Anda di halaman 1dari 24

Answers to

Self-Assessment Questions
Infectious Diseases I

HIV Infection and AIDS

(PeP). Which one of the following is the best plan


for this patient?

1. A 20-year-old man comes to the emergency department with a flu-like illness, rash (predominantly on
his trunk), fever, and pronounced lymphadenopathy.
He reports a sexual encounter 1 week prior with a male
partner of unknown HIV status. He reports being an
active smoker and intravenous drug user. Which one
of the following screening tests would be best to
perform in this patient?

A. Initiate two-drug antiretroviral therapy (ART) for


28 days.
B. Initiate three-drug ART for 72 hours; then deescalate to a two-drug regimen.
C. Perform rapid HIV test on the source person before ART initiation for proper regimen
selection.
D. Perform rapid HIV test and screening for hepatitis B virus (HBV) and hepatitis C virus (HCV),
but do not initiate ART.

A.
B.
C.
D.

CD4+ cell count.


Rapid HIV.
Enzyme-linked immunosorbent assay (ELISA).
HIV RNA.

2. Answer: A
Postexposure prophylaxis (PeP) should be offered
immediately after laboratory workup because this patient
presents within 72 hours (further within 36 hours) after
an act of anal intercourse (a high-risk behavior). Twodrug therapy with nucleoside reverse transcriptase inhibitors (NRTIs) is appropriate unless source data (e.g., known
HIV status and viral resistance) dictate a different approach
(Answer A is correct). Three-drug therapy has not proved
superior to two-drug therapy in PeP and may be associated
with more adverse drug events unless the source data are
known (Answer B is incorrect). Obtaining laboratory data
should not delay PeP initiation, especially when exposures
of less than 36 hours have been associated with improved
outcomes (Answer C is incorrect). The patient should be
offered PeP given the high-risk behavior and timing of
exposure (Answer D is incorrect).
1. Landovitz RJ, Currier JS. Postexposure prophylaxis for
HIV infection. N Engl J Med 2009;361:176875.
PubMed Link
2. Centers for Disease Control and Prevention. Antiretroviral postexposure prophylaxis after sexual, injection-drug use, or other nonoccupational exposure
to HIV in the United States. MMWR Recomm Rep
2005;54(RR-2):120.
PubMed Link

1. Answer: D

The HIV RNA viral load is the test of choice for identifying a patient with HIV during acute HIV syndrome
before antibody formation (Answer D is correct). CD4+
cell counts are not used as a definitive diagnosis of HIV
infection (Answer A is incorrect). Although a rapid HIV
test and enzyme-linked immunosorbent assay (ELISA)
may be performed clinically, given a syndrome consistent
with acute HIV, the patient might not have antibody formation for 26 weeks and thus would potentially have a falsenegative result (Answer B and Answer C are incorrect).
1. Greenwald JL, Burstein GR, Pincus J, Branson B. A rapid
review of rapid HIV antibody tests. Curr Infect Dis Rep
2006;8:12531.
PubMed Link
2. Self WH. Acute HIV infection: diagnosis and management in the emergency department. Emerg Med Clin
North Am 2010;28:38192.
PubMed Link
2. A 29-year-old HIV-negative man comes to the clinic
reporting receptive anal intercourse about 18 hours
ago. His partners HIV status was unknown. The
patient inquires about postexposure prophylaxis
PSAP-VII Infectious Diseases

Answers

4. A 36-year-old man has a CD4+ cell count of 165


cells/mm3 and HIV-1 RNA of 74,538 copies/mL. He
recently initiated ART with tenofovir/emtricitabine/
efavirenz. He has no known drug allergies, nor is he
taking any other drugs. He was noted to have a glucose-6-phosphate dehydrogenase (G6PD) deficiency.
Which one of the following would be best to add to
this patients regimen for prophylaxis?

3. A 28-year-old woman has had HIV infection for 5


years. She has decided to initiate ART because she and
her husband are going to attempt to have children. Her
current laboratory values include the following: CD4+
cell count 290 cells/mm3, HIV RNA 150,000 copies/
mL, white blood cell count (WBC) 3.1 x 103 cells/
mm3, hemoglobin 10.1 g/dL, aspartate aminotransferase (AST) 40 IU/mL, and alanine aminotransferase
(ALT) 36 IU/mL. She has no evidence of HCV infection, and she has received the hepatitis B vaccine. She
is an emergency medical technician working full-time
and reports no illicit drug use, smoking, or alcohol
abuse. Which one of the following regimens would
be the best initial choice for this patient?

A. Trimethoprim/sulfamethoxazole 160 mg/800


mg/day.
B. Dapsone 100 mg/day.
C. Atovaquone 1500 mg/day.
D. Azithromycin 1200 mg/week.
4. Answer: A
Trimethoprim/sulfamethoxazole is first-line therapy for Pneumocystis jiroveci pneumonia (PCP) prophylaxis, which is the only prophylaxis indicated in this patient
(Answer A is correct). Dapsone therapy is contraindicated
in glucose-6-phosphate dehydrogenase (G6PD)-deficient
patients because of increased risk of hemolytic anemia
(Answer B is incorrect). Atovaquone treatment is considered second-line treatment of PCP prophylaxis only if trimethoprim/sulfamethoxazole cannot be used (Answer C
is incorrect). Azithromycin does not provide protection
against PCP (Answer D is incorrect).
1. Youngster L, Arcavi L, Schechmaster R, Akayzen Y,
Popliski H, Shimonov J, et al. Medications and glucose6-phosphate dehydrogenase deficiency: an evidencebased review. Drug Saf 2010;33:71326.
PubMed Link
2. Centers for Disease Control and Prevention guidelines
for prevention and treatment of opportunistic infections in HIV-infected adults and adolescents. MMWR
Recomm Rep 2009;58(RR-4):1207.
PubMed Link

A. Once-daily fixed-dose abacavir/lamivudine and


twice-daily zidovudine.
B. Once-daily fixed-dose efavirenz/tenofovir/
emtricitabine.
C. Twice-daily fixed-dose zidovudine/lamivudine
and twice-daily nevirapine.
D. Once-daily fixed-dose tenofovir/emtricitabine
and lopinavir/ritonavir.
3. Answer: D
A once-daily fixed-dose combination of tenofovir/
emtricitabine and lopinavir/ritonavir is an alternative regimen, but it would be the best choice of those listed in a
patient preparing for pregnancy (Answer D is correct).
Once-daily fixed-dose combination abacavir/lamivudine
plus twice-daily zidovudine is incorrect because triple
nucleoside reverse transcriptase inhibitor (NRTI) therapy
is inferior to other standard regimens (Answer A is incorrect). Once-daily efavirenz/tenofovir/emtricitabine is a
first-line preferred regimen, but the patient is attempting
to become pregnant, and efavirenz is contraindicated in the
first trimester and should be avoided in patients attempting
to become pregnant (Answer B is incorrect). Twice-daily
fixed-dose combination zidovudine/lamivudine and twicedaily nevirapine is incorrect because the patients CD4+
count (290) puts her at increased risk of nevirapine-associated hepatotoxicity (Answer C is incorrect).
1. Torti C, Coastarelli S, De Silvestri A, Quiros-Roldan E,
Lapadula G, Cologni G, et al. Analysis of severe hepatic
events associated with nevirapine-containing regimens:
CD4+ T-cell count and gender in hepatitis C seropositive and seronegative persons. Drug Saf 2007;30:11619.
PubMed Link
2. Panel on Antiretroviral Guidelines for Adults and Adolescents. Guidelines for the use of antiretroviral agents in
HIV-1-infected adults and adolescents. Department of
Health and Human Services. October 14, 2011; 1167.
Available at aidsinfo.nih.gov/ContentFiles/AdultandAdolescentGL.pdf. Accessed April 9, 2012.

Answers

Questions 5 and 6 pertain to the following case.


M.M. is a 39-year-old woman with chronic active HBV and
HIV. She is seen in the clinic today and is eager to start treatment. Results of a complete metabolic panel are as follows:
sodium 139 mEq/L, potassium 3.9 mEq/L, blood urea
nitrogen (BUN) 15 mg/dL, and serum creatinine (SCr)
0.9 mg/dL; all other values were within normal limits. Her
CD4+ count is 45 cells/mm3, her HIV RNA is 106,500 copies/mL, and she is immunoglobulin G (IgG) positive for
toxoplasmosis. M.M. is known to be G6PD deficient. She
has steroid-induced osteoporosis and had a single episode
of oral thrush. Her current drugs include calcium 1500
mg/day plus vitamin D 400 international units/day. Documented allergies are to sulfa (rash) and chrysanthemums
(facial swelling).

PSAP-VII Infectious Diseases

5. Which one of the following is the best regimen for


treatment of M.M.s HIV-HBV coinfection?

G6PD deficiency, and although used clinically as monotherapy, it does not offer full protection against toxoplasmosis (Answer A is incorrect). Azithromycin, which is indicated for prophylaxis of MAC (CD4+ less than 50 cells/
mm3), should be included in the regimen. Acyclovir treatment in patients with herpes simplex is preferred because it
reduces herpes episodes and their potential impact on HIV
RNA, but it is not indicated because the patient has no history of herpes (Answer B is incorrect). Fluconazole is not
indicated as primary prophylaxis unless the patient has had
several recurrences of candidal infections. In addition, dapsone should be avoided in G6PD deficiency (Answer C is
incorrect).
1. Celum C, Wald A, Lingappa JR, Magaret AS, Wang RS,
Mugo N, et al. Acyclovir and transmission of HIV-1 from
persons infected with HIV-1 and HSV-2. N Engl J Med
2010;362:42739.
PubMed Link
2. Youngster L, Arcavi L, Schechmaster R, Akayzen Y,
Popliski H, Shimonov J, et al. Medications and glucose6-phosphate dehydrogenase deficiency: an evidencebased review. Drug Saf 2010;33:71326.
PubMed Link

A. Tenofovir/emtricitabine, darunavir, ritonavir, and


pegylated-interferon alfa (pegIFN).
B. Tenofovir/emtricitabine/efavirenz.
C. Abacavir/lamivudine, atazanavir, ritonavir, and
adefovir.
D. Zidovudine/lamivudine and raltegravir.
5. Answer: B
Tenofovir/emtricitabine/efavirenz is the preferred
first-line regimen in a patient coinfected with HIV-HBV
(hepatitis B virus) initiating therapy for both. The history
of osteoporosis may warrant more frequent bone mineral
density scans over time (Answer B is correct). Tenofovir plus emtricitabine would be first-line therapy for HBV;
thus, pegylated-interferon alfa (pegIFN) is not indicated
at this time (Answer A is incorrect). Although abacavir/
lamivudine, atazanavir plus ritonavir plus adefovir would
be appropriate to treat both disease states, it is not the preferred regimen (Answer C is incorrect). Zidovudine/lamivudine and raltegravir is not a preferred regimen for HIV.
Lamivudine monotherapy for HBV is not a preferred regimen and would require the addition of a second HBVactive drug (Answer D is incorrect).
1. Lok AS, McMahon BJ. Chronic hepatitis B: update 2009.
AASLD practice guidelines. Hepatology 2009;50:6612.
PubMed Link
2. Sax PE, Tierney C, Collier AC, Fischl MA, Mollan K,
Peeples L, et al. Abacavir-lamivudine versus tenofoviremtricitabine for initial HIV-1 therapy. N Engl J Med
2009;361:223040.
PubMed Link

7. A 25-year-old man comes to the clinic requesting preventive HIV therapy. He is currently single and discloses that he had four male partners in the past 6
months, with his last encounter 34 weeks ago. He uses
condoms for anal intercourse but rarely for oral intercourse. The patient has found several potential partners online and will be meeting all of them for a weekend getaway in 2 weeks. He has no specific medical history, he tested HIV negative 9 months ago, and he is
currently taking no long-term drugs. The patient did
experience the flu this past week but did not seek specific medical care. In addition to counseling on sexually transmitted diseases and proper condom use,
which one of the following would be best for this
patient?

6. Which one of the following would be best to initiate


for additional antimicrobial prophylaxis in M.M.?
A. Dapsone 100 mg/day, azithromycin 1200 mg
once weekly, and acyclovir 400 mg twice daily.
B. Atovaquone 1500 mg/day and acyclovir 400 mg
twice daily.
C. Dapsone 50 mg/day, pyrimethamine 75 mg/day,
azithromycin 1200 mg once weekly, fluconazole
100 mg/day, and acyclovir 400 mg twice daily.
D. Atovaquone 1500 mg/day and azithromycin 1200
mg once weekly.

A. Initiate tenofovir/emtricitabine once daily with


counseling on proper adherence.
B. Initiate darunavir/ritonavir once daily with counseling on proper adherence.
C. Perform rapid HIV test; if results are negative,
initiate tenofovir/emtricitabine once daily and
counsel on proper adherence.
D. Perform ELISA, obtain HIV RNA, and counsel
on avoiding high-risk behavior.

6. Answer: D

Atovaquone is indicated for PCP (CD4+ less than 200
cells/mm3) and toxoplasmosis (CD4+ less than 100 cells/
mm3 and positive immunoglobulin G [IgG]) prophylaxis
given sulfa allergy and G6PD deficiency. Azithromycin is
indicated for prophylaxis of Mycobacterium avium complex
(MAC) (CD4+ less than 50 cells/mm3; Answer D is correct). Dapsone should not be used in patients with a known
PSAP-VII Infectious Diseases

7. Answer: D
Both tests (ELISA and HIV RNA) should be ordered
and confirmed negative before initiating any preexposure prophylaxis (PrEP) (Answer D is correct). Preexposure prophylaxis with tenofovir/emtricitabine may be
3

Answers

(CAP) treatment with levofloxacin is appropriate (Answer


D is correct). A prednisone taper is not indicated in patients
with this womans respiratory status and apparent mild
disease with a Pao2 greater than 70 mm Hg. In addition,
empiric therapy should include treatment of CAP until
the diagnosis of PCP is confirmed (Answer A is incorrect).
Clindamycin and primaquine are second-line therapy, and
the patient has no contraindication to trimethoprim/sulfamethoxazole therapy (Answer B is incorrect). Atovaquone
is considered an alternative therapy, and dapsone does not
provide additional benefit to the regimen (Answer C is
incorrect).
1. Centers for Disease Control and Prevention. Guidelines
for prevention and treatment of opportunistic infections in HIV-infected adults and adolescents. MMWR
Recomm Rep 2009;58(RR-4):1207.
PubMed Link
2. Mandell LA, Wunderink RG, Anzueto A, Bartlett JG,
Campbell GD, Dean NC, et al. Infectious Diseases Society of America/American Thoracic Society consensus
guidelines on the management of community-acquired
pneumonia in adults. Clin Infect Dis 2007;44:S27S72.
PubMed Link

considered only after test results are known (Answer A is


incorrect). Darunavir/ritonavir has not been evaluated as
a potential option for PrEP (Answer B is incorrect). Given
that the patient had a flu-like illness 1 week prior and admits
to his last sexual encounter being 34 weeks ago, he would
be at high risk of acute HIV. A rapid HIV test might provide a false-negative test at this point in the disease process
(Answer C is incorrect).
1. Underhill K, Operario D, Skeer M, Mimiaga M, Mayer
K. Packaging PrEP to prevent HIV: an integrated framework to plan for pre-exposure prophylaxis implementation in clinical practice. J Acquir Immune Defic Syndr
2010;55:813.
PubMed Link
2. Grant RM, Lama JR, Anderson PL, McMahan V, Liu AY,
Vargas L, et al. Preexposure chemoprophylaxis for HIV
prevention in men who have sex with men. N Engl J Med
2010;363:258799.
PubMed Link
Questions 810 pertain to the following case.
G.J. is a 31-year-old woman (weight 65 kg, height 48
inches) admitted to the medical ward secondary to a compromised respiratory status. She reports a 2-week history
of cough and chest pain that has steadily worsened. G.J.s
platelet count is 123,000 cells/mm3, lactose dehydrogenase 400 IU/L, SCr 1.1 mg/dL, and BUN 24 mg/dL. Her
partial pressure of oxygen in arterial blood is 76 mm Hg.
G.J.s most recent CD4+ count was 41 cells/mm3, and she is
known to be IgG positive for toxoplasmosis. Chest radiography shows bilateral infiltrates with a ground-glass appearance. She is placed on supplemental oxygen with a 3L nasal
cannula; subsequently, her oxygen saturation is 98%. On
examination, it is also noted that she has esophageal candidiasis. Proper diagnostics are initiated.

9. Which one of the following is best for G.J.s esophageal candidiasis?


A.
B.
C.
D.

9. Answer: B

Fluconazole should be considered the most cost-effective option and is available for oral administration (Answer
B is correct). Although amphotericin B would work, the
risk-benefit ratio of this agent does not support its use as
first-line therapy (Answer A is incorrect). Nystatin should
not be used for esophageal disease (Answer C is incorrect).
Although micafungin is indicated, fluconazole should be
considered the more cost-effective treatment and offers an
oral option when the patient is able to swallow (Answer D
is incorrect).
1. Centers for Disease Control and Prevention. Guidelines
for prevention and treatment of opportunistic infections in HIV-infected adults and adolescents. MMWR
Recomm Rep 2009;58(RR-4):1207.
PubMed Link
2. Pappas PG, Kauffman CA, Andes D, Benjamin DK Jr,
Calandra TF, Edwards JE Jr, et al. Clinical practice guidelines for the management of candidiasis: 2009 updated
by the Infectious Diseases Society of America. Clin Infect
Dis 2009;48:50335.
PubMed Link

8. Which one of the following is the best empiric therapy for G.J.s pulmonary infection?
A. Trimethoprim/sulfamethoxazole 400 mg intravenously every 8 hours and prednisone 40 mg twice
daily to taper.
B. Levofloxacin 750 mg intravenously daily,
clindamycin 600 mg intravenously every 6 hours,
and primaquine 30 mg orally daily.
C. Atovaquone 750 mg orally twice daily, dapsone
100 mg/day, ceftriaxone 1 g intravenously daily,
and azithromycin 500 mg intravenously daily.
D. Trimethoprim/sulfamethoxazole 300 mg intravenously every 6 hours and levofloxacin 750 mg
intravenously daily.
8. Answer: D
Trimethoprim/sulfamethoxazole is first-line therapy
for PCP, and empiric community-acquired pneumonia
Answers

Amphotericin B 40 mg intravenously daily.


Fluconazole 200 mg intravenously daily.
Nystatin 100,000 units orally four times/day.
Micafungin 150 mg intravenously daily.

PSAP-VII Infectious Diseases

D. 800 mg once daily.

10. G.J. completes current antimicrobial treatment and


is being discharged after a 3-week hospitalization. A
repeat CD4+ count is 146 cells/mm3. For which one of
the following opportunistic infections (OIs) would
it be best for her to receive prophylaxis?

11. Answer: A
The dose of maraviroc should be reduced to 150 mg
twice daily because of the potent 3A4 inhibition from
boosted darunavir despite induction from etravirine
(Answer A is correct). Dosage must be decreased because
of potent 3A4 inhibition from boosted darunavir (Answer
B is incorrect). When given with both a 3A4 inducer and
inhibitor, the dosage should be decreased to 150 mg twice
daily. Dose increases are not appropriate in this patient
(Answer C and Answer D are incorrect).
1. Kakuda TN, Abel S, Davis J, Hamlin J, Scholler-Gyure M,
Mack R, et al. Pharmacokinetic interactions of maraviroc
with darunavir/ritonavir, maraviroc with etravirine, and
maraviroc with etravirine/darunavir/ritonavir in healthy
volunteers: results of two drug interaction trials. Antimicrob Agents Chemother 2011;55:22906. [E-pub 2011
Mar 7]
PubMed Link
2. Panel on Antiretroviral Guidelines for Adults and Adolescents. Guidelines for the use of antiretroviral agents in
HIV-1-infected adults and adolescents. Department of
Health and Human Services. October 14, 2011; 1167.
Available at aidsinfo.nih.gov/ContentFiles/AdultandAdolescentGL.pdf. Accessed April 9, 2012.

A. Mycobacterium avium complex (MAC), Pneumocystis jiroveci pneumonia (PCP), and


toxoplasmosis.
B. MAC and toxoplasmosis.
C. PCP only.
D. Candidiasis, MAC, and toxoplasmosis.
10. Answer: A
Having a CD4+ of less than 200 cells/mm3 is an indication for PCP prophylaxis, less than 100 cells/mm3 is an
indication for toxoplasmosis prophylaxis, and less than 50
cells/mm3 is an indication for MAC prophylaxis. Patients
CD4+ counts should be sustained for 3 months above
thresholds, and they should not experience an opportunistic infection (OI) in the interim to discontinue OI prophylaxis (Answer A is correct). The patient should also receive
prophylaxis for PCP after completion of treatment (Answer
B is incorrect). A CD4+ count of 41 cell/mm3 (less than 50
cells/mm3) requires prophylaxis for MAC. The patient is
IgG positive for toxoplasmosis as well, thus requiring prophylaxis (Answer C is incorrect). A single episode of candidiasis is not an indication for prophylaxis; PCP prophylaxis
is indicated (Answer D is incorrect).
1. Centers for Disease Control and Prevention. Guidelines
for prevention and treatment of opportunistic infections in HIV-infected adults and adolescents. MMWR
Recomm Rep 2009;58(RR-4):1207.
PubMed Link
2. Buchacz K, Baker RK, Palella FJ Jr, Lickhtenstein KA,
Chmiel JS, Novak RM, et al. AIDS-defining opportunistic illnesses in US patients, 1994-2007: a cohort study.
AIDS 2010;24:154959.
PubMed Link

12. A 24-year-old man (weight 68 kg) is admitted to the


inpatient ward with pancytopenia, prolonged diarrhea,
and overall fatigue and malaise. Initial blood cultures
and bone marrow aspirate are positive for Mycobacterium spp., later identified as MAC. He has a CD4+
count of 33 cells/mm3 and an HIV RNA of 35,350 copies/mL. His laboratory values include SCr 1.4 mg/dL
and BUN 34 mg/L. He has an allergy to sulfa (hives).
His last prescribed ART regimen included tenofovir/
emtricitabine/efavirenz, although he reports being
nonadherent. Which one of the following is best for
management of this patients OIs?
A. Azithromycin 600 mg, rifabutin 300 mg twice
daily, ethambutol 1200 mg/day, and atovaquone
1500 mg/day.
B. Clarithromycin 500 mg twice daily and dapsone
100 mg/day.
C. Atovaquone 1500 mg/day; defer MAC therapy
until CD4+ count is greater than 50 cells/mm3.
D. Azithromycin 1200 mg/week and dapsone 100
mg/day.

11. A 24-year-old woman with a history of nonadherence


has documented antiretroviral resistance. Her CD4+
count is 180 cells/mm3 and HIV RNA is 41,300 copies/mL. After tropism testing, it is determined she has
an R5-tropic virus. Her new regimen will include tenofovir/emtricitabine (3A4 neutral), darunavir and ritonavir (3A4 inhibitor), etravirine (3A4 inducer), and
maraviroc. She is also receiving omeprazole 20 mg/
day. Which one of the following maraviroc dosages
is best for this patient?

12. Answer: A
Starting two-drug therapy for MAC is appropriate
(clarithromycin and ethambutol), and prophylaxis for PCP
(atovaquone given sulfa allergy) is appropriate (Answer
A correct). Single-drug therapy for MAC is not optimal

A. 150 mg twice daily.


B. 300 mg twice daily.
C. 600 mg twice daily.
PSAP-VII Infectious Diseases

Answers

treatment (Answer B is incorrect). Single-drug therapy


with ethambutol for MAC is not proper treatment. Macrolide therapy is indicated in first-line regimens (Answer
C is incorrect). MAC prophylaxis (weekly azithromycin)
should not be used with active MAC infections (Answer D
is incorrect).
1. Centers for Disease Control and Prevention. Guidelines
for prevention and treatment of opportunistic infections in HIV-infected adults and adolescents. MMWR
2009;58:RR-4. Available at www.cdc.gov/mmwr/pdf/rr/
rr5804.pdf. Accessed April 9, 2012.
2. Panel on Antiretroviral Guidelines for Adults and Adolescents. Guidelines for the use of antiretroviral agents in
HIV-1-infected adults and adolescents. Department of
Health and Human Services. October 14, 2011; 1167.
Available at aidsinfo.nih.gov/ContentFiles/AdultandAdolescentGL.pdf. Accessed April 9, 2012.

ate with maintenance of immune reconstitution inflammatory syndrome (IRIS) with symptomatic care (Answer D
is correct). Antiretroviral therapy (ART) should be continued unless life-threatening symptoms occur (Answer A
and Answer C are incorrect). Although prednisone can be
used, it should be used only in the presence of significant
symptoms. There is no specific advantage presented to discontinue fluconazole (Answer B is incorrect).
1. Muller M, Wandel S, Colebunders R, Attia S, Furrer H,
Egger M; IeDEA Southern and Central Africa. Immune
reconstitution inflammatory syndrome in patients
starting antiretroviral therapy for HIV infection: a systematic review and meta-analysis. Lancet Infect Dis
2010;10:25161.
PubMed Link
2. French MA. HIV/AIDS: immune reconstitution
inflammatory syndrome: a reappraisal. Clin Infect Dis
2009;48:1017.
PubMed Link

13. A 31-year-old man was recently admitted to the medical ward with severe headache and photophobia. He
has a CD4+ count of 15 cells/mm3 and HIV RNA of
105,350 copies/mL. His current laboratory values
are SCr 1.0 mg/dL, BUN 17 mg/L, AST 25 IU/mL,
ALT 38 IU/mL, hemoglobin 10.3 g/dL, platelet count
167,000 cells/mm3, and WBC 2.9 x 103 cells/mm3.
He has no drug allergies. He reports being off antiretrovirals for several years and does not recall his previous regimen. A lumbar puncture is performed, and a
cerebrospinal fluid stain is positive for yeast. Given the
high suggestion of cryptococcal disease, amphotericin B 60 mg intravenously daily and flucytosine 1500
mg orally every 6 hours are initiated. After 2 weeks of
induction therapy, the patient is discharged to continue
fluconazole and is initiated on tenofovir/emtricitabine,
darunavir and ritonavir, and raltegravir with scheduled clinic follow-up. Three weeks after initiating ART,
he begins to experience fever, fatigue, and lymphadenopathy, prompting a visit to the emergency department. Given the concern for immune reconstitution inflammatory syndrome (IRIS), which one of
the following is best for this patient?

14. A nurse from the outpatient clinic reports to employee


health. She states she had a self-inflicted subcutaneous
puncture wound through her gloved hand 1 hour earlier after giving HBV vaccine to a known patient with
HIV infection. The needle was not bloody, although
the nurse bled from the puncture wound. The source
patients most recent laboratory values show a CD4+
count of 345 cells/mm3 and HIV RNA of less than
50 copies/mL, controlled on a regimen consisting of
tenofovir/emtricitabine and lopinavir/ritonavir. The
exposed nurse is very anxious and seeks advice on
proper PeP. Which one of the following is best for
this health care worker?
A. Initiate two-drug therapy for 28 days.
B. Initiate two-drug therapy pending rapid HIV testing on the nurse and discontinue if results are
negative.
C. Initiate three-drug therapy for 28 days.
D. PeP is not recommended at this time.

A. Discontinue ART and fluconazole and provide


symptomatic care as needed.
B. Continue ART, discontinue fluconazole, and initiate prednisone 60 mg/day.
C. Discontinue ART, initiate prednisone 60 mg/day,
and reinitiate induction therapy for cryptococcal
disease.
D. Continue ART and fluconazole and provide
symptomatic care as needed.

14. Answer: C
Postexposure prophylaxis should be initiated in a
patient with known exposure to an HIV-positive source
patient, although the risk of acquisition is very low in this
case. The regimen chosen should be consistent with the
source patients regimen. Although two-drug therapy may
be appropriate, given the patient has maintained viral suppression on a known regimen, the exposed patient should
receive the same regimen (i.e., three drugs) unless contraindicated (Answer C is correct). Although two-drug therapy may be appropriate, given the patient has maintained
viral suppression on a known regimen, the exposed patient
should receive the same regimen (i.e., three drugs) unless

13. Answer: D

Despite symptoms, therapy continuation is appropri-

Answers

PSAP-VII Infectious Diseases

contraindicated (Answer A is incorrect). A rapid HIV test


will likely not be positive because of delayed antibody formation and is not an indication to discontinue PeP (Answer
B is incorrect). For this case, PeP is recommended because
the patient is known to be HIV-infected (Answer D is
incorrect).
1. Landovitz RJ, Currier JS. Postexposure prophylaxis for
HIV infection. N Engl J Med 2009;361:176875.
PubMed Link
2. International AIDS Society-USA. Perspective occupational postexposure prophylaxis for HIV: the PEPline
perspective. Top HIV Med 2010;18:1747.
PubMed Link

16. A 34-year-old man with productive cough, night sweats,


pronounced lymphadenopathy, and a 15-pound weight
loss during the past month is admitted to the internal
medicine ward. He is placed on respiratory isolation
and receives a tuberculin skin test, chest radiography,
and sputum sample for acid-fact bacilli (AFB) smear,
and cultures are ordered. Chest radiography is highly
suggestive of granulomatous disease, and the AFB
smear returns positive. Four-drug therapy with isoniazid, rifampin, ethambutol, and pyrazinamide is initiated. Results of an HIV ELISA return reactive and are
later confirmed with Western blot. Laboratory results
show his CD4+ cell count is 168 cells/mm3, and his
HIV RNA is greater than 100,000 copies/mL. Which
one of the following is the best time to initiate ART
in this patient?

15. Your institution is a 300-bed community hospital in a


rural area of the Southeastern United States, which is a
high-prevalence area for HIV. Given the CDC recommendations for universal screening, hospital administration is requesting an update to the current policy for
testing in the emergency department. Which one of
the following would be the best testing approach?

A. After 1 week of Mycobacterium tuberculosis (MTB)


therapy.
B. After MTB therapy is narrowed to two-drug
therapy.
C. After MTB therapy is complete.
D. After 4 weeks of MTB therapy.

A. All patients aged 1364 should be routinely


screened after providing written consent to
testing.
B. High-risk patients should be screened at least
annually.
C. All patients aged 1364 should be routinely
screened with an approved antibody-based
screening test (e.g., ELISA or rapid test).
D. Patients presenting with symptoms consistent
with acute HIV syndrome or OI should be routinely screened (e.g., HIV RNA).

16. Answer: D
Antiretroviral therapy initiated within 28 weeks
of Mycobacterium tuberculosis (MTB)-targeted therapy
improves overall outcomes (Answer D is correct). To
reduce the risk of IRIS, the therapy should probably not
be initiated after 1 week of tuberculosis therapy (Answer A
is incorrect). Delays in therapy beyond 8 weeks have been
associated with worse clinical outcomes (Answer B and
Answer D are incorrect).
1. Velasco M, Castilla V, Sanz J, Gaspar G, Condes E, Barros
C, et al. Effect of simultaneous use of highly active antiretroviral therapy on survival of HIV patients with tuberculosis. J Acquir Immune Defic Syndr 2009;50:14852.
PubMed Link
2. Abdool Karim SS, Naidoo K, Grobler A, Padayatchi N,
Baxter C, Gray A, et al. Timing of initiation of antiretroviral drugs during tuberculosis therapy. N Engl J Med
2010;362:697706.
PubMed Link

15. Answer: C
The 2006 CDC guidelines recommend that all
patients aged 1364 be screened through an opt-out process (Answer C is correct). No specific written consent is
required, according to federal guidelines. State guidelines
should be consulted for varied levels of required documentation (Answer A is incorrect). Risk-based testing has not
been shown effective and is not recommended (Answer B
and Answer D are incorrect).
1. Centers for Disease Control and Prevention. Revised
recommendations for HIV testing of adults, adolescents,
and pregnant women in health-care settings. MMWR
Recomm Rep 2006;55(RR-14):117.
PubMed Link
2. National HIV/AIDS Clinicians Consultation Center.
2011 Compendium of State HIV Testing Laws. Available
at www.nccc.ucsf.edu/consultation_library/state_hiv_
testing_laws. Accessed April 9, 2012.

PSAP-VII Infectious Diseases

17. A 54-year-old man receives a diagnosis of HIV infection and, 6 weeks later, presents to the clinic to begin
ART. His baseline laboratory values are CD4+ count
388 cells/mm3, HIV RNA 121,000 copies/mL, SCr
0.7 mg/dL, BUN 20 mg/L, AST 45 IU/mL, ALT 28
IU/mL, hemoglobin 11.3 g/dL, platelet count 337,000
cells/mm3, and WBC 8.9 x 103 cells/mm3. He has a
documented allergy to sulfa (rash). His current daily
regimen includes hydrochlorothiazide 25 mg, esomeprazole 40 mg, and pravastatin 80 mg. Which one of
7

Answers

the following ART regimens would be best for this


patient?
A.
B.
C.
D.

ART errors (Answer C is incorrect). The use of tenofovir


might contribute to errors in very specific instances (e.g.,
significant renal impairment), but it is not the most likely
risk in this patient (Answer D is incorrect).
1. Pastakia SD, Corbett AH, Raasch RH, Napravnik S, Correll TA. Frequency of HIV-related medication errors and
associated risk factors in hospitalized patients. Ann Pharmacother 2008;42:4917.
PubMed Link
2. Mok S, Minson Q. Drug-related problems in hospitalized
patients with HIV infection. Am J Health Syst Pharm
2008;65:559.
PubMed Link

Tenofovir/emtricitabine, atazanavir, and ritonavir.


Zidovudine/lamivudine and rilpivirine.
Tenofovir/emtricitabine and raltegravir.
Abacavir/lamivudine and efavirenz.

17. Answer: C
Tenofovir/emtricitabine and raltegravir is a first-line
treatment option (Answer C is correct). Atazanavir should
not be used with esomeprazole 40 mg because of impaired
absorption of atazanavir because of changes in gastric pH
(Answer A is incorrect). Rilpivirine with zidovudine/lamivudine is not a preferred option in treatment-naive patients
(Answer B is incorrect). Abacavir/lamivudine is not a preferred NRTI backbone (Answer D is incorrect).
1. Khanlou H, Farthing C. Co-administration of atazanavir
with proton pump inhibitors and H2 blockers. J Acquir
Immune Defic Syndr 2005;39:503.
PubMed Link
2. Panel on Antiretroviral Guidelines for Adults and Adolescents. Guidelines for the use of antiretroviral agents in
HIV-1-infected adults and adolescents. Department of
Health and Human Services. October 14, 2011; 1167.
Available at aidsinfo.nih.gov/ContentFiles/AdultandAdolescentGL.pdf. Accessed April 9, 2012.

19. A 45-year-old man is seen in the clinic today with an


apparent case of oral candidiasis. He reports being
completely adherent to his current regimen of zidovudine/lamivudine, darunavir, and ritonavir. For the past
2 months, his CD4+ has declined to 285 cells/mm3
(from 350 cells/mm3), and his HIV RNA has increased
to 1750 copies/mL (previously undetectable). Which
one of the following would be the most feasible way to
evaluate adherence in this patient?
A. Perform therapeutic drug monitoring for darunavir serum concentrations.
B. Obtain a complete blood cell count and refill history at pharmacy.
C. Obtain a complete metabolic panel and refill history at pharmacy.
D. Ask the patient to bring drug bottles to the next
clinic visit and perform a pill count.

18. After total hip replacement for avascular necrosis, a


38-year-old man is transferred to the orthopedic ward.
He has a history of HIV infection and has been successfully controlled (CD4+ 419 cells/mm3, HIV RNA
less than 50 copies/mL) on an ART regimen of tenofovir/emtricitabine/efavirenz. He is placed on a morphine patient-controlled analgesia pump and docusate. While rounding on the antimicrobial stewardship
team, you proactively review his regimen and ensure
the appropriate regimen is reinitiated. Which one of
the following risk factors for ART error in hospitalized patients will most likely be encountered in this
patient?
A.
B.
C.
D.

19. Answer: B
Almost 100% of patients on zidovudine therapy will
develop a macrocytic anemia with elevated mean corpuscular volume. Obtaining a refill history is always prudent to evaluate outpatient adherence (Answer B is correct). Therapeutic drug monitoring may not indicate sustained adherence and is not readily available in most institutions (Answer A is incorrect). Adherence to this regimen
could not be sufficiently evaluated with a metabolic panel
(Answer C is incorrect). Although a pill count could be
done, it might not provide accurate information (Answer D
is incorrect).
1. Romanelli F, Empey K, Pomeroy C. Macrocytosis as
an indicator of medication (zidovudine) adherence in
patients with HIV infection. AIDS Patient Care STDS
2002;16:40511.
PubMed Link
2. Panel on Antiretroviral Guidelines for Adults and Adolescents. Guidelines for the use of antiretroviral agents in
HIV-1-infected adults and adolescents. Department of
Health and Human Services. October 14, 2011; 1167.

Coformulated products not on formulary.


Concurrent hepatic dysfunction.
Surgery.
Tenofovir as part of ART regimen.

18. Answer: A
Nonformulary drugs are a major risk factor for antiretroviral errors in the hospital, especially in combination
products. This often leads to incomplete regimens or delays
in therapy reinitiation (Answer A is correct). Concurrent
liver dysfunction would not be a specific risk factor for antiretroviral errors for this regimen (Answer B is incorrect).
Surgery might predispose the patient to the possibility of
missed doses, but it is not the most likely risk factor for
Answers

PSAP-VII Infectious Diseases

Available at aidsinfo.nih.gov/ContentFiles/AdultandAdolescentGL.pdf. Accessed April 9, 2012.

A. Tenofovir/emtricitabine/efavirenz.
B. Zidovudine/lamivudine/atazanavir/ritonavir.
C. Abacavir/lamivudine/lopinavir/ritonavir.
D. Tenofovir/emtricitabine/raltegravir.

20. A 23-year-old woman comes to the sexually transmitted diseases clinic inquiring about measures to help
prevent HIV acquisition. She admits to a monogamous
relationship with a known HIV-positive man. She
acknowledges that they use condoms regularly, and she
is tested every 6 months as recommended by her primary care physician. Which one of the following is
most appropriate for this patient?

21. Answer: D
In HIV-positive patients with risk factors for cardiovascular disease (CVD), care should be taken to ensure the
selection of an antiretroviral regimen that minimizes further CVD risk. Because several agents can adversely affect
serum lipids as well as contribute to the development of
metabolic disorders including lipodystrophy, agents not
linked to these effects should be chosen preferentially. The
selection of a combination regimen that includes tenofovir, emtricitabine, and raltegravir can minimize the development of lipid abnormalities and metabolic disorders
(Answer D is correct), whereas the selection of a regimen
that includes zidovudine can lead to insulin resistance and
lipodystrophy (Answer B is incorrect). In addition, the use
of a protease inhibitor (PI)-based regimen, particularly
one that includes lopinavir/ritonavir, can adversely affect a
patients serum lipid profile (Answer C is incorrect). Moreover, the patient is currently receiving simvastatin, which is
contraindicated in patients receiving PIs because of a significant drug interaction. Although the regimen of tenofovir, emtricitabine, and efavirenz does not adversely affect
lipid profiles to the same extent as PIs, its effects on raising low-density lipoprotein (LDL) cholesterol are greater
than the regimen of tenofovir, emtricitabine, and raltegravir
(Answer A is incorrect).
1. Panel on Antiretroviral Guidelines for Adults and Adolescents. Guidelines for the use of antiretroviral agents in
HIV-1-infected adults and adolescents. Department of
Health and Human Services. October 14, 2011; 1167.
Available at aidsinfo.nih.gov/ContentFiles/AdultandAdolescentGL.pdf. Accessed April 9, 2012.
2. Lennox JL, DeJesus E, Berger DS, Lazzarin A, Pollard
RB, Ramalho Madruga JV, et al. Raltegravir versus efavirenz regimens in treatment-naive HIV-1-infected
patients: 96-week efficacy, durability, subgroup, safety,
and metabolic analysis. J Acquir Immune Defic Syndr
2010;55:3948.
PubMed Link

A. Start tenofovir/emtricitabine once daily and provide adherence counseling.


B. Start applying nonoxynol-9 gel before each sexual
encounter.
C. Obtain partners ART regimen and resistance
profile to determine appropriate prophylactic
regimen.
D. Encourage continued appropriate condom use
and partners adherence to ART.
20. Answer: D
In this patient, condom use is an effective means of
prevention, especially if the infected partner is well controlled with maximal viral suppression (Answer D is correct). Oral PrEP has not been studied extensively in women
and thus cannot be recommended (Answer A is incorrect).
Non-HIV microbicides have shown inconsistent results for
preventing HIV and should not be recommended (Answer
B is incorrect). Preexposure prophylaxis regimens have not
been determined in this manner and should not be recommended (Answer C is incorrect).
1. Centers for Disease Control and Prevention. Interim
guidance: preexposure prophylaxis for the prevention of
HIV infection in men who have sex with men. MMWR
2011;60:658.
PubMed Link
2. Ramjee G, Kamali A, McCormack S. The last decade of
microbicide clinical trials in Africa: from hypothesis to
facts. AIDS 2010;24(suppl 4):S40S49.
PubMed Link

Primary Care in HIV

22. A 54-year-old man with diabetes and HIV is successfully being treated with tenofovir/emtricitabine/
darunavir/ritonavir. After 3 months of dietary and lifestyle changes, his lipid profile is as follows: total cholesterol, 259 mg/dL; low-density lipoprotein (LDL) cholesterol, 167 mg/dL; high-density lipoprotein (HDL)
cholesterol, 37 mg/dL; and triglycerides, 280 mg/dL.
Which one of the following would best manage this
patients hyperlipidemia at this time?

21. A 52-year-old man with type 2 diabetes mellitus (DM)


and hyperlipidemia treated with simvastatin recently
received a diagnosis of HIV infection and will soon be
starting antiretroviral therapy (ART). His virus is sensitive to all antiretroviral agents. Which one of the following ART regimens is best for this patient?
PSAP-VII Infectious Diseases

Answers

A. Replace tenofovir and emtricitabine with abacavir


and lamivudine.
B. Replace darunavir and ritonavir with unboosted
atazanavir.
C. Initiate lipid-lowering therapy with atorvastatin
10 mg/day.
D. Initiate lipid-lowering therapy with pravastatin 20
mg/day.

legs, consistent with lipoatrophy. His CD4+ cell count is


489 cells/mm3, and his viral load is undetectable. He currently receives ART consisting of lamivudine, zidovudine,
and lopinavir boosted by ritonavir. This is the only ART
regimen he has received.
23. For which one of the following comorbid conditions would it be best for A.L. to receive diagnostic
testing?

22. Answer: C
When managing hyperlipidemia in HIV-positive
patients who are currently receiving antiretroviral therapy
(ART), options consist of switching antiretroviral agents
or providing lipid-lowering therapy. In this case, the patient
is a receiving a preferred antiretroviral regimen (tenofovir/emtricitabine/darunavir/ritonavir) that has minimal
adverse lipid effects and to which his HIV has responded
successfully. Lipid-lowering therapy is therefore appropriate, and atorvastatin is the agent of choice (Answer C is correct). Tenofovir and emtricitabine have not been associated
with hyperlipidemia; therefore, switching these agents to
abacavir and lamivudine would not be useful (Answer A is
incorrect). Although changing this patient to an unboosted
PI would remove the adverse lipid effects of ritonavir, atazanavir must be boosted when combined with tenofovir
(Answer B is incorrect). When choosing a statin, pravastatin is often a safe and effective agent. However, there is
a significant drug interaction between pravastatin and
darunavir; therefore, low-dose atorvastatin is a safer and
potentially more effective option for this patient (Answer
D is incorrect).
1. Panel on Antiretroviral Guidelines for Adults and Adolescents. Guidelines for the use of antiretroviral agents in
HIV-1-infected adults and adolescents. Department of
Health and Human Services. October 14, 2011; 1167.
Available at aidsinfo.nih.gov/ContentFiles/AdultandAdolescentGL.pdf. Accessed April 9, 2012.
2. Calza L, Manfredi R, Colangeli V, Tampellini L, Sebastiani T, Pocaterra D, et al. Substitution of nevirapine or
efavirenz for protease inhibitor versus lipid-lowering
therapy for the management of dyslipidaemia. AIDS
2005;19:10518.
PubMed Link
3. Sekar VJ, Spinosa-Guzman S, Marien K, et al. Pharmacokinetic drug-drug interaction between the new HIV
protease inhibitor darunavir (TMC114) and the lipidlowering agent pravastatin. 8th International Workshop
on Clinical Pharmacology of HIV Therapy. April 1618,
2007, Budapest, Hungary. Abstract 54.

A. Type 2 DM and hyperlipidemia.


B. Hyperlipidemia and osteopenia.
C. Chronic kidney disease (CKD) and insulin
resistance.
D. Vitamin D deficiency and osteoporosis.
23. Answer: A

This patient has lipoatrophy, and he has been receiving
an antiretroviral regimen with agents known to be associated with insulin resistance and hyperlipidemia (Answer A
is correct). Among the other choices, diagnostic testing for
bone disorders is not indicated in this individual because he
is younger than 50 years and has no history of fragility fracture (Answer B and Answer D and incorrect). Answer C is
also incorrect because lipoatrophy has not been associated
with chronic kidney disease (CKD).
1. Moreno S, Miralles C, Negredo E, Domingo P, Estrada
V, Gutierrez F, et al. Disorders of body fat distribution in
HIV-1-infected patients. AIDS Rev 2009;11:12634.
PubMed Link
2. Wohl DA, Brown TT. Management of morphologic
changes associated with antiretroviral use in HIV-infected
patients. J Acquir Immune Defic Syndr 2008;49(suppl
2):S93S100.
PubMed Link
24. In addition to the screening above, which one of the
following would best manage A.L.s lipodystrophy?
A. Change ART to tenofovir/emtricitabine plus atazanavir and ritonavir.
B. Change lopinavir plus ritonavir to darunavir plus
ritonavir.
C. Maintain current ART regimen.
D. Provide therapy with tesamorelin.
24. Answer: A
Changing the therapy for this patient might improve
comorbid conditions such as hyperlipidemia and insulin
resistance and might provide a partial reversal of lipoatrophy.
The switch should include replacing zidovudine with tenofovir and lopinavir with atazanavir or darunavir (Answer A
is correct). Replacing lopinavir alone with darunavir might
improve hyperlipidemia but might not affect lipoatrophy

Questions 23 and 24 pertain to the following case.


A.L. is a 48-year-old man who received a diagnosis of HIV
infection in 1996. Today, on physical examination, generalized subcutaneous fat loss is noted in the face, arms, and
Answers

10

PSAP-VII Infectious Diseases

26. The following patients have HIV infection. In which


one would dual-energy x-ray absorptiometry be
most indicated to evaluate for the presence of
osteoporosis?

symptoms (Answer B is incorrect). Maintaining the current regimen is also not optimal because lipoatrophy symptoms would remain, and insulin resistance might be present
(Answer C is incorrect). Finally, treatment with tesamorelin is indicated only for the management of visceral fat accumulation, not lipoatrophy (Answer D is incorrect).
1. Moreno S, Miralles C, Negredo E, Domingo P, Estrada
V, Gutierrez F, et al. Disorders of body fat distribution in
HIV-1-infected patients. AIDS Rev 2009;11:12634.
PubMed Link
2. Wohl DA, Brown TT. Management of morphologic
changes associated with antiretroviral use in HIV-infected
patients. J Acquir Immune Defic Syndr 2008;49(suppl
2):S93S100.
PubMed Link

A.
B.
C.
D.

26. Answer: C
Infection with HIV should be considered a risk factor
for premature bone loss. Therefore, these patients should fit
into the category of earlier dual-energy x-ray absorptiometry (DXA) screening consisting of postmenopausal women
and men 50 years and older (Answer C is correct). Furthermore, in women with HIV who are younger than 50 or 50
years or older but premenopausal, DXA screening is not
required (Answer A is incorrect). In those with HIV infections who are younger than 50 and have not had a previous
fragility fracture, DXA screening is also not required, even
in the presence of other risk factors like hypogonadism
(Answer B is incorrect) or vitamin D deficiency (Answer D
is incorrect).
1. McComsey GA, Tebas P, Shane E, Yin MT, Overton ET,
Huang JS, et al. Bone disease in HIV infection: a practical review and recommendations for HIV care providers.
Clin Infect Dis 2010;51:93746.
PubMed Link
2. Foundation NO. Clinicians Guide to Prevention and
Treatment of Osteoporosis. Washington, DC: National
Osteoporosis Foundation, 2010.

25. A 35-year-old man with HIV infection has a history of


anal condylomata, with two or three recurrences in the
past 5 years. He recently received an anal Papanicolaou
(Pap) smear test that indicated the presence of lowgrade squamous intraepithelial lesions. Which one of
the following is the most appropriate next step in managing this patients abnormal test results?
A.
B.
C.
D.

A follow-up Pap smear in 1 year.


A follow-up Pap smear in 6 months.
Testing for human papillomavirus infection.
Anoscopy testing and possible biopsy.

25. Answer: D
This patient shows low-grade changes on anal PaPap
smear testing. Although definitive recommendations are
unavailable, most authorities agree that, if there are any
abnormal findings on Pap smear, all patients should receive
high-resolution anoscopy with subsequent biopsy of any
irregular areas (Answer D is correct). This is because even
with low-grade lesions found on Pap smear, patients can still
have a significant risk (46% to 56%) of high-grade lesions
or intraepithelial neoplasia on biopsy. Follow-up Pap smear
testing is not appropriate until anoscopy and biopsy are
performed (Answer A and Answer B are incorrect). Finally,
the role of HPV (human papillomavirus) testing in this setting is not well defined; therefore, it is not recommended
for this patient (Answer C is incorrect).
1. New York State Department of Health. Neoplastic Complications of HIV Infection. July 2007:133. Available at
www.hivguidelines.org/clinical-guidelines/adults/neoplastic-complications-of-hiv-infection/. Accessed April 9, 2012.
2. Chin-Hong PV, Palefsky JM. Natural history and clinical management of anal human papillomavirus disease
in men and women infected with human immunodeficiency virus. Clin Infect Dis 2002;35:112734.
PubMed Link

PSAP-VII Infectious Diseases

A 48-year-old premenopausal woman.


A 45-year-old man with hypogonadism.
A 56-year-old postmenopausal woman.
A 41-year-old man with vitamin D deficiency.

27. A 45-year-old man with an extensive ART history presents with a CD4+ cell count of 89 cells/mm3 and signs
of oral thrush. He states that he is aware he requires
HIV treatment but that he is fearful of the adverse
effects, particularly those consistent with lipodystrophy that he has experienced with treatment in the past.
Which one of the following ART regimens was this
patient most likely treated with in the past?
A.
B.
C.
D.

Abacavir, lamivudine, and efavirenz.


Darunavir/ritonavir and raltegravir.
Zidovudine, lamivudine, and lopinavir/ritonavir.
Tenofovir, emtricitabine, and nevirapine.

27. Answer: C
Of the regimens listed in this question, the one that
includes a thymidine analog (zidovudine and stavudine)
is most likely to be associated with lipodystrophy-related
adverse effects (Answer C is correct). Although lipodystrophy can occur with other ART agents, of the options listed,
it is much less likely to occur with abacavir, lamivudine, and
11

Answers

efavirenz (Answer A is incorrect), darunavir/ritonavir and


raltegravir (Answer B is incorrect), or tenofovir, emtricitabine, and nevirapine (Answer D is incorrect).
1. Moreno S, Miralles C, Negredo E, Domingo P, Estrada
V, Gutierrez F, et al. Disorders of body fat distribution in
HIV-1-infected patients. AIDS Rev 2009;11:12634.
PubMed Link
2. Wohl DA, Brown TT. Management of morphologic
changes associated with antiretroviral use in HIV-infected
patients. J Acquir Immune Defic Syndr 2008;49(suppl
2):S93S100.
PubMed Link

29. A study evaluates the 10-year risk of coronary heart


disease (CHD) in men and women infected with HIV.
Compared with patients receiving protease inhibitors
(PIs), patients who are naive to therapy have an odds
ratio of 0.57 (95% confidence interval [CI], 0.360.89)
and patients receiving nonPI-based regimens have an
odds ratio of 0.74 (95% CI, 0.531.01). Former ART
users have an odds ratio of 0.68 (95% CI, 0.461.03).
Which one of the following best describes these
results?
A. Patients naive to ART have a lower risk of developing CHD than do patients receiving PIs.
B. Former users of ART have a lower risk of developing CHD than do patients receiving nonPIbased regimens.
C. Patients receiving nonPI-based regimens have
a lower risk of developing CHD than do patients
receiving PIs.
D. Patients receiving PIs have a greater risk of developing CHD than do former users of ART.

28. In which one of the following patients with HIV


infection would it be best to consider diagnostic
testing for HIV-associated nephropathy?
A. A newly infected 34-year-old African American
man with a serum creatinine (SCr) of 1.2 mg/dL
and glucosuria, but no proteinuria.
B. A 45-year-old African American man receiving a
diagnosis 10 years ago with an SCr of 1.5 mg/dL
and proteinuria, but no glucosuria.
C. A newly infected 38-year-old white man with an
SCr of 1.2 mg/dL and glucosuria and proteinuria.
D. A 21-year-old white man receiving a diagnosis 3
years ago with an SCr of 1.4 mg/dL but no glucosuria or proteinuria.

29. Answer: A

With an odds ratio of 0.57 and a 95% confidence interval that does not include the null value of 1, the patients who
are naive to therapy have a lower risk of coronary heart disease (CHD) than those receiving PIs (Answer A is correct).
Those receiving nonPI-based regimens and patients who
are former antiretroviral recipients both have odds ratios
that may indicate a lower risk of CHD. However, the 95%
confidence intervals for these values include the null value
of 1. Therefore, it is possible that the risk of CHD is similar in patients receiving PIs and non-PI regimens as well as
those who are former users of ART (Answer B, Answer C,
and Answer D are incorrect).
1. Kaplan RC, Kingsley LA, Sharrett AR, Li X, Lazar J,
Tien PC, et al. Ten-year predicted coronary heart disease
risk in HIV-infected men and women. Clin Infect Dis
2007;45:107481.
PubMed Link
2. Law MG, Friis-Moller N, El-Sadr WM, R, Reiss P,
DArminio Monforte A, et al. The use of the Framingham equation to predict myocardial infarctions in HIVinfected patients: comparison with observed events in
the D:A:D Study. HIV Med 2006;7:21830.
PubMed Link

28. Answer: B

Human immunodeficiency virusassociated nephropathy (HIVAN) is usually seen in African American patients
who are not newly infected and will most often present with
an elevated serum creatinine (SCr) and mild to severe proteinuria (Answer B is correct). It is much less likely to be
present in a newly HIV-positive patient with a normal SCr
(Answer A and Answer C are incorrect) or in a white man
with a lack of proteinuria or glucosuria present on urinalysis
(Answer D is incorrect). A definitive diagnosis of HIVAN
requires kidney biopsy because this presentation can overlap with other glomerular diseases.
1. Gupta SK, Eustace JA, Winston JA, Boydston II, Ahuja
TS, Rodriguez RA, et al. Guidelines for the management
of chronic kidney disease in HIV-infected patients: recommendations of the HIV Medicine Association of the
Infectious Diseases Society of America. Clin Infect Dis
2005;40:155985.
PubMed Link
2. Fine DM, Perazella MA, Lucas GM, Atta MG.
Renal disease in patients with HIV infection. Drugs
2008;68:96380.
PubMed Link

Answers

Questions 30 and 31 pertain to the following case.


B.T. is a 48-year-old African American man with newly
diagnosed type 2 DM, controlled hypertension, and a baseline SCr of 1.4 mg/dL. He was recently initiated on tenofovir, emtricitabine, and atazanavir/ritonavir for HIV management. About 10 months later on a routine visit, his SCr
12

PSAP-VII Infectious Diseases

concentration is 1.8 mg/dL and his urinalysis shows 2+


protein, 1+ blood, and 1+ glucose.

and prednisone, although it plays a role in managing HIVassociated nephropathy, is not helpful in patients with tenofovir toxicity (Answer D is incorrect).
1. Gupta SK, Eustace JA, Winston JA, Boydston II, Ahuja
TS, Rodriguez RA, et al. Guidelines for the management
of chronic kidney disease in HIV-infected patients: recommendations of the HIV Medicine Association of the
Infectious Diseases Society of America. Clin Infect Dis
2005;40:155985.
PubMed Link
2. Goicoechea M, Liu S, Best B, Sun S, Jain S, Kemper C,
et al. Greater tenofovir-associated renal function decline
with protease inhibitor-based versus nonnucleoside
reverse-transcriptase inhibitor-based therapy. J Infect Dis
2008;197:1028.
PubMed Link

30. Which one of the following best describes B.T.s


kidney dysfunction?
A.
B.
C.
D.

HIV-associated nephropathy.
Hypertension-associated CKD.
Tenofovir-associated nephrotoxicity.
Diabetic nephropathy.

30. Answer: C

This patient most likely has tenofovir-associated nephrotoxicity (Answer C is correct). The onset of this patients
decline in kidney function is temporally associated with initiating tenofovir and likely does not represent CKD associated with hypertension (this patient also has controlled
hypertension) or diabetes (he has new-onset diabetes)
(Answer B and Answer D are incorrect). Furthermore,
disease onset occurred in proximity to the patients starting tenofovir. It also presented with a urinalysis with mild
hematuria and glucosuria, which is uncharacteristic of HIVassociated nephropathy (Answer A is incorrect).
1. Gupta SK, Eustace JA, Winston JA, Boydston II, Ahuja
TS, Rodriguez RA, et al. Guidelines for the management
of chronic kidney disease in HIV-infected patients: recommendations of the HIV Medicine Association of the
Infectious Diseases Society of America. Clin Infect Dis
2005;40:155985.
PubMed Link
2. Fine DM, Perazella MA, Lucas GM, Atta MG.
Renal disease in patients with HIV infection. Drugs
2008;68:96380.
PubMed Link

32. Compared with patients receiving continuous ART,


patients randomized to treatment interruption in the
SMART trial had a hazard ratio of 1.49 (95% CI, 1.04
2.11) for experiencing a major cardiovascular disease (CVD) event. Which one of the following best
describes these results?
A. Patients receiving continuous ART had a 49%
greater risk of experiencing a CVD event than did
patients randomized to treatment interruption.
B. The risk of experiencing a CVD event in the treatment interruption group was as much as 2 times
greater than in the continuous ART group.
C. There is no statistically significant difference
between study groups for the risk of experiencing
a CVD event.
D. Patients randomized to treatment interruption
had 1.5 times more CVD events than did patients
receiving continuous ART.

31. Which one of the following represents the best


ART option for B.T.?

32. Answer: B
A hazard ratio of 1.49 with a 95% confidence interval
of 1.042.11 can be interpreted as the treatment interruption groups having a risk of CVD as much as 2 times greater
than the continuous ART group (Answer B is correct). An
incorrect interpretation would be that the continuous ART
group had a greater risk of CVD (Answer A is incorrect). In
addition, because the confidence interval does not include
1, there was a statistically significant difference between
groups (Answer C is incorrect). Finally, a hazard ratio of
1.49 does not indicate a 150% increase in the total number
of events for the treatment interruption group (Answer D is
incorrect). Instead, it represents a 50% increase.
1. Strategies for Management of Antiretroviral Therapy
(SMART) Study Group; Emery S, Neuhaus JA, Phillips AN, Babiker A, Cohen CJ, Gatell JM, et al. Major
clinical outcomes in antiretroviral therapy (ART)-naive

A. All ART should be discontinued until kidney


function recovers.
B. Tenofovir should be discontinued and replaced
with alternative ART.
C. Continue current ART with dose adjusted for creatinine clearance.
D. Continue antiretrovirals and add prednisone therapy until kidney function recovers.
31. Answer: B
In patients with tenofovir-associated nephrotoxicity,
it is best to remove tenofovir and replace it with another
agent (Answer B is correct). Renal dose adjustments alone
will not reverse tenofovir toxicity; rather, they will continue
to expose the patient to this agent (Answer C is incorrect).
Removing all ART is not necessary (Answer A is incorrect),
PSAP-VII Infectious Diseases

13

Answers

participants and in those not receiving ART at baselines


in the SMART study. J Infect Dis 2008;197:113344.
PubMed Link
2. Strategies for Management of Antiretroviral Therapy
(SMART) Study Group; Lundgren JD, Babiker A, ElSadr W, Emery S, Grund B, Neaton JD, et al. Inferior clinical outcome of the CD4+ cell count-guided antiretroviral treatment interruption strategy in the SMART study:
role of CD4+ cell counts and HIV RNA concentrations
during follow-up. J Infect Dis 2008;197:114555.
PubMed Link

of additional ART options 5 years later, T.Y.s regimen was


changed to zidovudine/lamivudine and lopinavir/ritonavir. Since then, his HIV-1 RNA has remained suppressed,
and his CD4+ cell count has been stable at 450500 cells/
mm3. The patient is moderately overweight and has some
abdominal fat accumulation, facial wasting, and loss of leg
fat. Laboratory values are as follows: SCr 1.1 mg/dL, fasting blood glucose 138 mg/dL, total cholesterol 210 mg/dL,
LDL cholesterol 162 mg/dL, HDL cholesterol 35 mg/dL,
and triglycerides 220 mg/dL.
34. Which one of the following most accurately
describes the reason for the uncertainty of the diagnosis of diabetes for T.Y.?

33. A 41-year-old man given a diagnosis of HIV in 1989


received zidovudine monotherapy for 5 years before
being told to discontinue this agent. He has not
received treatment since then and is now referred with
biopsy-proven HIV-associated nephropathy. Which
one of the following is best to recommend for this
patient?

A. An oral glucose tolerance test is required for


definitive diagnosis in patients with HIV.
B. Fasting blood glucose concentrations may be
inaccurate in HIV-positive patients.
C. Hemoglobin A1C (A1C) testing is the preferred
method for diagnosis in HIV-positive patients.
D. A repeat fasting blood glucose concentration of
125 mcg/dL or greater is required for diagnosis.

A. ART and an angiotensin-converting enzyme


(ACE) inhibitor.
B. ART and prednisone.
C. ACE inhibitor and prednisone.
D. ART only.

34. Answer: D
This patient has a single fasting blood glucose concentration greater than 125 mg/dL, requiring a repeat test
with similar results for a diagnosis of diabetes. The diagnosis of diabetes in patients with HIV infection should be performed using previously recommended strategies including abnormal fasting serum blood glucose concentrations
(125 mg/dL or greater) observed on two separate occasions or using a glucose tolerance test (Answer D is correct). Newer recommendations that allow the use of hemoglobin A1C (A1C) testing for diabetes diagnosis in the general population should not be used at this time in patients
with HIV infection given the inaccuracies in the results of
this test in this population (Answer C is incorrect). An oral
glucose tolerance test can be used for diagnosing diabetes
in patients with HIV infection but is not required (Answer
A is incorrect), and fasting blood glucose concentrations
should remain accurate in these patients despite inaccuracies in A1C measurements (Answer B is incorrect).
1. Kim PS, Woods C, Georgoff P, Crum D, Rosenberg A,
Smith M, et al. A1C underestimates glycemia in HIV
infection. Diabetes Care 2009;32:15913.
PubMed Link
2. American Diabetes Association. Standards of medical
care in diabetes 2010. Diabetes Care 2010;33(suppl
1):S11S61.
PubMed Link

33. Answer: D
The most appropriate initial option for the management of this condition in a patient who is not currently
receiving ART is the initiation of ART alone (Answer D is
correct). The initiation of ART improves the prognosis of
HIVAN and is considered to provide the best outcomes,
given the direct role of the HIV virus in the pathogenesis of
this condition. Although prednisone and angiotensin-converting enzyme (ACE) inhibitors have a role in the treatment of HIVAN, they are not considered first-line options
(Answer A, Answer B, and Answer C are incorrect).
1. Gupta SK, Eustace JA, Winston JA, Boydston II, Ahuja
TS, Rodriguez RA, et al. Guidelines for the management
of chronic kidney disease in HIV-infected patients: recommendations of the HIV Medicine Association of the
Infectious Diseases Society of America. Clin Infect Dis
2005;40:155985.
PubMed Link
2. Fine DM, Perazella MA, Lucas GM, Atta MG.
Renal disease in patients with HIV infection. Drugs
2008;68:96380.
PubMed Link
Questions 34 and 35 pertain to the following case.
T.Y. is a 48-year-old man who received stavudine, lamivudine, and nelfinavir as his initial ART regimen at the time of
HIV diagnosis 12 years ago. He achieved virologic suppression on this regimen; however, because of the availability
Answers

35. Which one of the following daily statin regimens


would be best to initiate in T.Y.?
14

PSAP-VII Infectious Diseases

A.
B.
C.
D.

Atorvastatin 20 mg.
Rosuvastatin 10 mg.
Simvastatin 20 mg.
Lovastatin 40 mg.

completed should the use of calcium and vitamin D supplementation or the application of bisphosphonate therapy be
considered (Answer B and Answer C are incorrect). Many
of these secondary causes can be common in HIV-positive
patients and include the following: low testosterone in men,
adrenal insufficiency, CKD, and poor nutrition. In addition,
although tenofovir has been associated with increased bone
loss in patients with HIV infection, there is no evidence that
discontinuing this agent will prevent a fracture (Answer D
is incorrect).
1. McComsey GA, Tebas P, Shane E, Yin MT, Overton ET,
Huang JS, et al. Bone disease in HIV infection: a practical review and recommendations for HIV care providers.
Clin Infect Dis 2010;51:93746.
PubMed Link
2. Mondy K, Yarasheski K, Powderly WG, Whyte M, Claxton S, DeMarco D, et al. Longitudinal evolution of bone
mineral density and bone markers in human immunodeficiency virus-infected individuals. Clin Infect Dis
2003;36:48290.
PubMed Link

35. Answer: B
Of the options available, rosuvastatin 10 mg represents the best choice (Answer B is correct). When selecting
rosuvastatin or atorvastatin, it is important to begin therapy with the lowest dose of the agent when given in combination with PIs to minimize potential adverse effects, even
when a patient requires a significant LDL reduction. Therefore, initiating therapy with atorvastatin 20 mg is not recommended (Answer A is incorrect). In addition, the patient
is currently receiving a PI, which would have a significant
interaction with simvastatin (Answer C is incorrect) or lovastatin (Answer D is incorrect).
1. Panel on Antiretroviral Guidelines for Adults and Adolescents. Guidelines for the use of antiretroviral agents in
HIV-1-infected adults and adolescents. Department of
Health and Human Services. October 14, 2011; 1167.
Available at aidsinfo.nih.gov/ContentFiles/AdultandAdolescentGL.pdf. Accessed April 9, 2012.
2. Grundy SM, Cleeman JI, Merz CN, Brewer HB Jr, Clark
LT, Hunninghake DB, et al. National Heart Lung and
Blood Institute; American College of Cardiology Foundation; American Heart Association Implications of
recent clinical trials for the National Cholesterol Education Program Adult Treatment Panel III guidelines. Circulation 2004;110:22739.
PubMed Link
3. Singh S, Willig JH, Mugavero MJ, Crane PK, Harrington
RD, Knopp RH, et al. Comparative effectiveness and toxicity of statins among HIV-infected patients. Clin Infect
Dis 2011;52:38795.
PubMed Link

37. Which one of the following statements best de-scribes


nonAIDS-defining cancers (NADCs)?
A. The incidence of NADCs decreased after ART
was developed.
B. ART reduced the overall incidence of anal cancer,
but not lung cancer, in the HIV population.
C. The risk of NADCs has been linked to low CD4+
cell counts.
D. Anal cancer, lung cancer, and non-Hodgkin lymphoma represent most NADCs.
37. Answer: C
One of the essential differences between AIDS-defining malignancies (ADMs) and NADCs is that ADMs
require the patient to have profound immunodeficiency.
However, although immunodeficiency is not a prerequisite for NADCs, they have been linked to low CD4 cell
counts (Answer C is correct). In addition, the use of ART
has changed the prevalence of neoplastic diseases in the
HIV population. After the introduction of ART, the incidence of ADMs declined, whereas the incidence of NADCs
increased (Answer A is incorrect). Furthermore, the incidence of both lung cancer and anal cancer increased,
despite the use of ART, and together with Hodgkin lymphoma, these cancers make up most NADCs currently
diagnosed in HIV-positive patients (Answer B and Answer
D are incorrect).
1. New York State Department of Health. Neoplastic Complications of HIV Infection. July 2007:133. Available at
www.hivguidelines.org/clinical-guidelines/adults/neoplastic-complications-of-hiv-infection/. Accessed April 9, 2012.

36. Which one of the following would be best to recommend for an HIV-positive patient recently given a
diagnosis of osteoporosis?
A. Assess for secondary causes of osteoporosis.
B. Initiate supplementation with calcium and vitamin D.
C. Initiate pharmacologic therapy with zoledronic
acid.
D. Discontinue tenofovir therapy if it is part of the
patients ART regimen.
36. Answer: A
The first step in assessing an HIV-positive patient
with osteoporosis is evaluating for the presence of secondary causes (Answer A is correct). Only after this step is

PSAP-VII Infectious Diseases

15

Answers

2. Hessol NA, Pipkin S, Schwarcz S, Cress RD, Bacchetti


P, Scheer S, et al. The impact of highly active antiretroviral therapy on non-AIDS-defining cancers among adults
with AIDS. Am J Epidemiol 2007;165:114353.
PubMed Link
3. Shiels MS, Pfeiffer RM, Gail MH, Hall HI, Li J, Chaturvedi
AK, et al. Cancer burden in the HIV-infected population
in the United States. J Natl Cancer Inst 2011;103:75362.
PubMed Link

1. Panel on Antiretroviral Guidelines for Adults and Adolescents. Guidelines for the use of antiretroviral agents in
HIV-1-infected adults and adolescents. Department of
Health and Human Services. October 14, 2011; 1167.
Available at aidsinfo.nih.gov/ContentFiles/AdultandAdolescentGL.pdf. Accessed April 9, 2012.
2. Calza L, Manfredi R, Chiodo F. Statins and fibrates
for the treatment of hyperlipidaemia in HIV-infected
patients receiving HAART. AIDS 2003;17:8519.
PubMed Link

38. A 47-year-old postmenopausal woman given a diagnosis of HIV in 1994 has an extensive history of ART. She
has been clinically stable during the past 2 years on her
current regimen (tenofovir, lamivudine, and lopinavir/
ritonavir); her current CD4+ cell count is 280 cells/
mm3 and her viral load is undetectable. Her total cholesterol is 350 mg/dL, HDL cholesterol is 35 mg/dL,
and triglycerides are 600 mg/dL. Her LDL cholesterol
could not be calculated because of her high triglyceride concentration. Which one of the following would
best manage this patients dyslipidemia?
A. Switch the antiretroviral regimen to tenofovir,
lamivudine, and atazanavir/ritonavir.
B. Switch lopinavir/ritonavir to darunavir/ritonavir,
and add lipid-lowering therapy with pravastatin
and fenofibrate.
C. Maintain the antiretroviral regimen, and recommend diet, exercise, and lifestyle modification.
D. Maintain the antiretroviral regimen, and add
lipid-lowering therapy with pravastatin and
fenofibrate.

Questions 39 and 40 pertain to the following case.


K.B. is a 49-year-old African American man with HIV infection managed with tenofovir, emtricitabine, and efavirenz.
He has a family history of DM and reports excessive thirst
and an unintentional weight loss of 10 lb (5 kg) in the past
year. His viral load is undetectable, CD4+ cell count is 460
cells/mm3, fasting blood glucose is 205 mg/dL, and SCr is
1.6 mg/dL.
39. Which one of the following would be best for K.B.?
A. Repeat the fasting blood glucose.
B. Test his A1C.
C. Perform an oral glucose tolerance test.
D. Diagnose DM.
39. Answer: D
A patient who has obvious symptoms of hyperglycemia, including unintentional weight loss and excessive
thirst with an elevated fasting blood glucose, can be given
a diagnosis of diabetes without further testing (Answer D
is correct). Therefore, additional testing is unnecessary,
including repeat fasting blood glucose (Answer A) is incorrect, A1C test (Answer B is incorrect), or oral glucose tolerance test (Answer C is incorrect).
1. American Diabetes Association. Standards of medical
care in diabetes 2010. Diabetes Care 2010;33(suppl
1):S11S61.
PubMed Link
2. Howard AA, Floris-Moore M, Lo Y, Arnsten JH, Fleischer
N, Klein RS. Abnormal glucose metabolism among older
men with or at risk for HIV. HIV Med 2006;7:38996.
PubMed Link

38. Answer: D
This patient has dyslipidemia, including hypertriglyceridemia that requires intervention. The elevations in total
cholesterol and triglycerides observed in this patient are
likely the result of several factors, including the presence of
lopinavir and ritonavir. In this case, the patient has extensive antiretroviral experience and is currently clinically stable on her present regimen. Given her history, it may be
very difficult to change agents and ensure virologic efficacy.
As a result, the best option for this patient is to maintain her
current antiretroviral regimen while initiating lipid-lowering therapy (Answer D is correct). Switching these agents
to another regimen is a possibility, although the impact
of this intervention would be less than that observed with
providing lipid-lowering pharmacotherapy (Answer A and
Answer B are incorrect). In addition, the drug-drug interaction between pravastatin and darunavir should be avoided
(Answer B is incorrect). Finally, given the degree of dyslipidemia present in this patient, diet, exercise, and lifestyle
modification alone would not affect the lipid profile to the
desired extent (Answer C is incorrect).

Answers

40. Which one of the following therapeutic interventions would be best for K.B.?
A. Initiate metformin and lifestyle interventions.
B. Initiate glipizide and lifestyle interventions.
C. Change efavirenz to darunavir/ritonavir.
D. Change tenofovir/emtricitabine to lamivudine/
zidovudine.

16

PSAP-VII Infectious Diseases

40. Answer: B
This patient should be treated with an oral glucoselowering medication, in this case glipizide, in addition to
lifestyle interventions (Answer B is correct). This patients
elevated SCr is a contraindication to receiving metformin;
otherwise, this agent would be preferred over glipizide as
initial therapy (Answer A is incorrect). The patients current antiretroviral regimen has been virologically successful, and it carries a very low risk of diabetes and glucose
intolerance as opposed to other agents. As a result, changing the patients regimen is not indicated and would not
improve his diabetes diagnosis (Answer C and Answer D
are incorrect).
1. Nathan DM, Buse JB, Davidson MB, Ferrannini E, Holman RR, Sherwin R, et al. Medical management of
hyperglycemia in type 2 diabetes: a consensus algorithm
for the initiation and adjustment of therapya consensus statement of the American Diabetes Association and
the European Association for the study of diabetes. Diabetes Care 2009;32:193203.
PubMed Link
2. Adeyemi O, Vibhakar S, Max B. Are we meeting the American Diabetes Association goals for HIV-infected patients
with diabetes mellitus? Clin Infect Dis 2009;49:799802.
PubMed Link

ethambutol are not the best option (Answer A and Answer


D are incorrect). The combination of ethionamide and clofazimine represents two drugs of last resort and is likely to
be less effective and more toxic than ethambutol and streptomycin (Answer C is incorrect).
1. American Thoracic Society/Centers for Disease Control/Infectious Diseases Society of America. Treatment of tuberculosis. Am J Respir Crit Care Med
2003;167:60362.
PubMed Link
2. World Health Organization. 2010. Treatment of Tuberculosis: Guidelines, 4th ed. Available at http://whqlibdoc.who.int/publications/2010/9789241547833_eng.pdf.
Accessed April 9, 2012.
42. A 46-year-old woman (height 57, weight 54 kg) is taking rifampin 600 mg, isoniazid 300 mg, pyrazinamide
1000 mg, and ethambutol 800 mg/day for 2 months.
She reports having blurry vision and difficulty with
color differentiation. Her most recent acid-fast bacillus
(AFB) culture is negative. Her aspartate transaminase
(AST) and alanine transaminase (ALT) today are 42
IU/L and 46 IU/L, respectively. She has hypothyroidism, for which she is taking levothyroxine 25 mcg/day;
and she is allergic to penicillin (rash). Which one of the
following is the best option for this patient?
A. Discontinue pyrazinamide and ethambutol.
B. Discontinue ethambutol.
C. Reduce the dose of pyrazinamide to 800 mg.
D. Reduce the dose of ethambutol to 600 mg.

Tuberculosis
41. A 62-year-old woman has been treated for 3 months
with isoniazid, rifampin, and pyrazinamide using
directly observed therapy (DOT). The sputum collected at the end of 2 months is culture positive for
Mycobacterium tuberculosis (MTB). Clinically, the
patient is still symptomatic, with cough and occasional
night sweats, and chest radiography has not changed
from 3 months earlier. In addition to continuing current therapy, which one of the following is the best
option to add for this patient?
A. Streptomycin.
B. Ethambutol and streptomycin.
C. Ethionamide and clofazimine.
D. Ethambutol.

42. Answer: A
Ethambutol is associated with retrobulbar neuritis.
Patients may report changes in visual acuity, an inability to
see the color green, or both. Ethambutol should be discontinued in this case. Because the patient received 2 months
of pyrazinamide and is culture negative, pyrazinamide also
should be discontinued (Answer A is correct). The patient
is experiencing these adverse effects at an ethambutol dose
of 15 mg/kg; lowering the dose could result in lower-thandesired serum concentrations and irreversible ocular damage (Answer D is incorrect). Because there is no need to
continue pyrazinamide, a dose reduction or discontinuation of ethambutol alone would be inappropriate (Answer
C and Answer B are incorrect).
1. Yee D, Valiquette C, Pelletier M, Parisien I, Rocher I,
Menzies D. Incidence of serious side effects from first-line
antituberculosis drugs among patients treated for active
tuberculosis. Am J Respir Crit Care Med 2003;167:1472.
PubMed Link
2. Peloquin CA. Therapeutic drug monitoring in the treatment of tuberculosis. Drugs 2002;62:216983.
PubMed Link

41. Answer: B
First-line drugs for treatment of active tuberculosis
(TB) include isoniazid, rifampin, pyrazinamide, and ethambutol. The patient is not responding to therapy, as evidenced by her still being culture positive at 2 months
and clinically still symptomatic while receiving isoniazid,
rifampin, and pyrazinamide. Adding a combination of ethambutol and streptomycin to the current therapy is the best
choice (Answer B is correct). Because a single drug should
never be added to a failing regimen, streptomycin and

PSAP-VII Infectious Diseases

17

Answers

In addition to counseling on alcohol consumption,


which one of the following is best for this patient?

43. Two months ago, a 36-year-old emergency department


nurse had an exposure at work from a patient with a
positive sputum for MTB. The nurses purified protein
derivative (PPD) test was read as 9 mm of induration 2
days ago. He has no symptoms such as cough or weight
loss, and chest radiography is normal. Prior PPD tests
showed no induration. The nurse has diabetes and
rheumatoid arthritis and no known drug allergies. His
drugs include metformin 500 mg twice daily and prednisone 20 mg once daily. Which one of the following
is the best therapeutic plan for this patient?

A.
B.
C.
D.

44. Answer: A
About 10% of infected patients develop reactivation
disease, and the greatest risk is within 2 years of infection.
The main issue is that the index case is from an area of high
isoniazid resistance (i.e., Philippines). Therefore, rifampin
might be a better choice, at least until susceptibility tests
can be performed for the index case (Answer A is correct).
Because of resistance rates, isoniazid is not the best option
in this case (Answer B is incorrect). Rifampin and pyrazinamide are no longer recommended, and ethambutol and
pyrazinamide would be used only for cases of suspected
multidrug-resistant (MDR) TB exposure (Answer C and
Answer D are incorrect).
1. Saukkonen JJ, Cohn DL, Jasmer RM, Schenker S, Jereb
JA, Nolan CM, et al. An official ATS statement: hepatotoxicity of antituberculosis therapy. Am J Respir Crit
Care Med 2006;174:935.
PubMed Link
2. Yee D, Valiquette C, Pelletier M, Parisient I, Rocher I,
Manzies D. Incidence of serious side effects from first-line
antituberculosis drugs among patients treated for active
tuberculosis. Am J Respir Crit Care Med 2003;167:1472.
PubMed Link

A. Repeat PPD in another 2 weeks.


B. Order an interferon-gamma release assay (IGRA)
for confirmation.
C. Start isoniazid daily for 9 months.
D. Start isoniazid and rifampin daily for 2 months.
43. Answer: C
Around 10% of infected patients develop reactivation disease. The greatest risk of developing an infection
is within 2 years of infection. A positive purified protein
derivative (PPD) in a recent contact of a patient with TB
is considered positive if the reaction is 5 mm of induration
or more. Isoniazid given for 9 months is the preferred drug
for treating latent TB infection (Answer C is correct). Starting several drugs is not required in this case (Answer D is
incorrect). Interferon-gamma release assay (IGRA) would
be inappropriate because the PPD test can boost IGRA
response and should be performed no more than 3 days
after PPD is placed (Answer B is incorrect). There is no
need to repeat a PPD because the current test is positive
(Answer A is incorrect).
1. Mazurek GH, Jereb J, Vernon A, LoBue P, Goldberg S,
Castro K, et al. Updated guidelines for using interferon
gamma release assays to detect Mycobacterium tuberculosis infection United States, 2010. MMWR Recomm
Rep 2010;59(RR-5):125.
PubMed Link
2. American Thoracic Society/Centers for Disease Control and Prevention. Targeted tuberculin skin testing and
treatment of latent tuberculosis infection. Am J Respir
Crit Care Med 2000;161:S221S247.
PubMed Link

45. A 30-year-old woman is taking isoniazid for a recent


TB exposure. She has been on therapy for 2 months.
Her AST and ALT today are 30 IU/L and 36 IU/L,
respectively. On initiation of isoniazid, her AST and
ALT values were 10 IU/L and 12 IU/L, respectively.
Which one of the following plans is most appropriate for this patient?
A. Continue isoniazid and repeat liver enzymes in
24 weeks.
B. Discontinue isoniazid until liver enzymes return
to baseline.
C. Discontinue isoniazid and repeat liver enzymes in
24 weeks.
D. Discontinue isoniazid and start rifampin and
pyrazinamide.

44. Four weeks ago, a 50-year-old respiratory therapist was


exposed in the intensive care unit to a patient from the
Philippines with tuberculosis (TB). Susceptibility data
are pending for that patients isolate. The therapists
PPD test last year was negative. His current PPD is
read as 8 mm of induration; he has no symptoms, and
chest radiography is normal. His baseline liver function tests show an AST of 102 IU/L and an ALT of 62
IU/L, both caused by chronic alcohol consumption.
Answers

Rifampin daily for 4 months.


Isoniazid daily for 9 months.
Rifampin and pyrazinamide for 2 months.
Ethambutol and pyrazinamide for 6 months.

45. Answer: A
The criteria for discontinuing isoniazid for suspected
liver toxicity is 3 times the upper limit of normal (ULN)
with symptoms or 5 times the ULN without symptoms.
The patients liver function tests have increased to about 3
times baseline since therapy initiation. The patient is not
symptomatic; therefore, isoniazid can be continued with
18

PSAP-VII Infectious Diseases

repeat liver enzymes in 24 weeks (Answer A is correct).


The patients liver enzymes should be monitored at least
monthly, and she should be observed for potential liver toxicity. Discontinuing treatment or changing to a different
regimen, thus depriving the patient of the benefit without
sufficient cause, is not indicated (Answer B, Answer C, and
Answer D are incorrect).
1. American Thoracic Society/Centers for Disease Control/Infectious Diseases Society of America. Treatment of tuberculosis. Am J Respir Crit Care Med
2003;167:60362.
PubMed Link
2. Saukkonen JJ, Cohn DL, Jasmer RM, Schenker S, Jereb
JA, Nolan CM, et al. An official ATS statement: hepatotoxicity of antituberculosis therapy. Am J Respir Crit
Care Med 2006;174:935.
PubMed Link

Treatment of tuberculosis. Am J Respir Crit Care Med


2003;167:60362.
PubMed Link
Questions 47 and 48 pertain to the following case.
M.T. is a 30-year-old man who traveled to Tibet. Now,
4 weeks later, he presents to his physician with cough,
fever, weight loss, and night sweats. M.T. has depression,
for which he is taking sertraline 50 mg/day. He takes no
other drugs and is allergic to sulfa (rash). Current laboratory values include serum creatinine (SCr) 1.0 mg/dL,
AST 42 IU/L, and ALT 46 IU/L. An initial sputum sample is AFB positive.
47. Which one of the following is best to initiate for
M.T.?
A. Isoniazid, rifampin, ethambutol, and
pyrazinamide.
B. Isoniazid, rifampin, and ethambutol.
C. Isoniazid, rifabutin, ethambutol, and
pyrazinamide.
D. Isoniazid and rifampin.

46. A 68-year-old Asian man with active TB has been taking rifampin, isoniazid, pyrazinamide, and ethambutol
for 5 weeks. He reports that his right big toe has been
painful for 2 weeks, and recently, he has had difficulty
walking around the house. On examination, the right
big toe is tender and red. His current a uric acid level
is 9 mg/dL compared with a baseline of 4 mg/dL at
the beginning of therapy. Which one of the following
plans is most appropriate for this patient?
A.
B.
C.
D.

47. Answer: A
Because of the relatively high proportion of adult
patients with TB caused by organisms that are resistant to
isoniazid, four drugs are necessary in the initial phase for
the 6-month regimen to be maximally effective. In most
circumstances, the treatment regimen for all adults with
previously untreated TB should consist of a minimum of
2 months of isoniazid, rifampin, pyrazinamide, and ethambutol until susceptibility results are available (Answer
A is correct). Starting a two- or three-drug regimen would
be inappropriate because susceptibilities are not available
yet (Answer B and Answer D are incorrect). The patient
is not taking any drugs that would require the substitution
of rifampin with rifabutin to minimize drug interactions
(Answer C is incorrect).
1. American Thoracic Society/Centers for Disease Control/Infectious Diseases Society of America. Treatment of tuberculosis. Am J Respir Crit Care Med
2003;167:60362.
PubMed Link
2. Caminero JA. Treatment of multidrug-resistant tuberculosis: evidence and controversies. Int J Tuberc Lung Dis
2006;10:82937.
PubMed Link

Discontinue isoniazid and add moxifloxacin.


Discontinue rifampin and add moxifloxacin.
Discontinue pyrazinamide only.
Discontinue ethambutol only.

46. Answer: C
Hyperuricemia may occur in patients receiving pyrazinamide, but acute gout is uncommon. Pyrazinamide
causes uric acid retention. The patients baseline uric acid
concentration was normal. Repeat testing is recommended
if a gouty attack occurs, as in this case. Pyrazinamide therapy should be discontinued because this patient has developed acute gouty arthritis with hyperuricemia (Answer C
is correct). Changing isoniazid or rifampin to moxifloxacin
will not affect the uric acid (Answer A and Answer B are
incorrect). Although ethambutol can cause increases in uric
acid levels, pyrazinamide is more likely to be the cause of a
gouty attack (Answer D is incorrect).
1. Yee D, Valiquette C, Pelletier M, Parisient I, Rocher I,
Manzies D. Incidence of serious side effects from first-line
antituberculosis drugs among patients treated for active
tuberculosis. Am J Respir Crit Care Med 2003;167:1472.
PubMed Link
2. American Thoracic Society/Centers for Disease
Control/Infectious Diseases Society of America.

PSAP-VII Infectious Diseases

48. Two months later, the TB susceptibility data reveal


that M.T.s strain of MTB is resistant to isoniazid and
rifampin. Which one of the following is best for
M.T.?
A. Repeat smear before any regimen changes.
19

Answers

B. Add streptomycin to current regimen.


C. Discontinue isoniazid and rifampin, and begin
streptomycin and cycloserine.
D. Discontinue isoniazid and rifampin, and begin
streptomycin and levofloxacin.

Department of Health and Human Services. Atlanta:


CDC, 2010.
2. Akolo C, Adetifa I, Shepperd S, Volmink J. Treatment
of latent tuberculosis infection in HIV infected persons.
Cochrane Database Syst Rev 2010;1:CD000171.
PubMed Link

48. Answer: D
Multidrug-resistant TB, by definition, is resistant to
both isoniazid and rifampin but may be resistant to additional drugs as well. Discontinuing isoniazid and rifampin
and beginning streptomycin and levofloxacin provides two
bactericidal drugs, with levofloxacin being the next best
drug to add in this situation (Answer D is correct). For this
case, waiting for a longer time with the incorrect regimen
is not appropriate (Answer A is incorrect). Adding streptomycin is incorrect because it adds a single drug to an inadequate regimen (Answer B is incorrect). Discontinuing isoniazid and rifampin and beginning streptomycin and cycloserine could be used in a more extreme case of drug resistance, but it is not needed in this case (Answer C is incorrect). This option also introduces a fairly toxic drug, cycloserine, before it is essential to do so.
1. American Thoracic Society/Centers for Disease Control/Infectious Diseases Society of America. Treatment of tuberculosis. Am J Respir Crit Care Med
2003;167:60362.
PubMed Link
2. World Health Organization. 2011. WHO Guidelines
for the Programmatic Management of Drug Resistant
tuberculosis: 2011 Update. Available at http://whqlibdoc.who.int/publications/2011/9789241501583_eng.pdf.
Accessed April 9, 2012.

50. A 26-year-old woman presents to her personal physician with a cough, fatigue, and night sweats for 2 weeks.
She works as an emergency department nurse and
reports having been exposed to TB about 3 months
ago. Her PPD is read at 12 mm; QuantiFERON and
AFB are positive. She is 4 months pregnant. Which
one of the following regimens would be best for this
patient?
A.
B.
C.
D.

50. Answer: B
Treatment with isoniazid, rifampin, and ethambutol appears to be the safest option with supporting data
for this pregnant woman (Answer B is correct). Treatment with isoniazid, rifampin, and pyrazinamide is used
in some countries, but it is not recommended currently
by the CDC because information on pyrazinamide safety
is lacking for pregnant women (Answer A is incorrect).
Treatment with isoniazid, ethambutol, and pyrazinamide
deprives the patient of rifamycin, one of the most important drugs (Answer C is incorrect). Treatment with isoniazid, rifampin, and streptomycin introduces streptomycin,
which is avoided in pregnant women because of toxicity
(Answer D is incorrect).
1. Mnyani CN, McIntyre JA. Tuberculosis in pregnancy.
BJOG 2011;118:22631.
PubMed Link
2. Hamadeh MA, Glassroth J. Tuberculosis and pregnancy.
Chest 1992;101:111420.
PubMed Link

49. Which one of the following individuals has the


greatest risk of developing active TB if infected?
A. Low-income homemaker with diabetes of 10
years duration, on insulin.
B. Long-term cigarette smoker currently on chemotherapy for lung cancer.
C. Homeless man with HIV infection for 2 years.
D. Nurse working in a New York City hospital with
recent positive PPD.

51. A 23-year-old Hispanic man with HIV and active


TB infections is receiving antiretroviral therapy with
emtricitabine, tenofovir, and atazanavir/ritonavir. His
anti-TB regimen consists of rifabutin, isoniazid, pyrazinamide, and ethambutol by DOT. He reports that his
right eye has hurt for 3 days and is now red. Which one
of the following would be best to discontinue in this
patient?

49. Answer: C
All factors mentioned (diabetes, cancer chemotherapy, HIV, and recent positive PPD) are risk factors for progression to active disease after infection. Of all of these factors, HIV infection is by far the greatest risk. The risk of TB
in an HIV-positive person is about 10% per year (Answer
C is correct) compared with a 10% lifetime risk of most
other patients (Answer A, Answer B, and Answer D are
incorrect).
1. Centers for Disease Control and Prevention (CDC).
Reported Tuberculosis in the United States, 2009, U.S.

Answers

Isoniazid, rifampin, and pyrazinamide.


Isoniazid, rifampin, and ethambutol.
Isoniazid, ethambutol, and pyrazinamide.
Isoniazid, rifampin, and streptomycin.

A. Ethambutol.
B. Isoniazid.
C. Pyrazinamide.
D. Rifabutin.
20

PSAP-VII Infectious Diseases

need to be extended to 9 months, not12 months, because


of her cavitary lesion (Answer C is incorrect).
1. Dorman SE, Johnson JL, Goldberg S, Muzanye G, Padayatchi N, Bozeman L, et al. Substitution of moxifloxacin for
isoniazid during intensive phase treatment of pulmonary
tuberculosis. Am J Respir Crit Care Med 2009;180:273.
PubMed Link
2. Peloquin CA. Therapeutic drug monitoring in the treatment of tuberculosis. Drugs 2002;62:216983.
PubMed Link

51. Answer: D
Rifabutin can cause uveitis, which typically causes
the eye to become painful and red and should be temporarily discontinued in this case (Answer D is correct). In
HIV-positive patients taking ART as well as being treated
for TB, rifabutin is often used instead of rifampin because
of drug interactions. Once resolved, rifabutin can be reinitiated at a lower dose, but this may run the risk of an inadequate dose and selecting for acquired rifamycin resistance.
Ethambutol-induced optic neuritis is not painful (Answer
A is incorrect). Isoniazid peripheral neuropathy does not
present with eye symptoms (Answer B is incorrect). Pyrazinamide is not known to cause optic neuritis (Answer C is
incorrect).
1. Centers for Disease Control and Prevention. Managing Drug Interactions in the Treatment of HIV-Related
Tuberculosis. Available at www.cdc.gov/tb/publications/guidelines/TB_HIV_Drugs/default.htm.
Accessed April 9, 2012.
2. Centers for Disease Control and Prevention. Guidelines
for prevention and treatment of opportunistic infections in HIV-infected adults and adolescents. MMWR
2009;58:RR-4. Available at www.cdc.gov/mmwr/pdf/rr/
rr5804.pdf. Accessed April 9, 2012.

53. A 32-year-old HIV-positive man recently received a


diagnosis of pulmonary TB. He has been on antiretroviral therapy of lopinavir/ritonavir, zidovudine, and
lamivudine for 3 years and remains stable. His CD4+
count is 550 cells/mm3. His primary care provider initiated isoniazid, rifampin, pyrazinamide, and ethambutol 2 weeks ago. The patient returns to the infectious
diseases clinic for his follow-up appointment. In addition to monitoring response to TB, CD4+ count,
and HIV viral load, which one of the following
plans is most appropriate for this patient?
A.
B.
C.
D.

52. A 68-year-old woman has received 7 weeks of therapy


with rifampin, isoniazid, and pyrazinamide. Today, she
returns to the clinic and reports increased cough and
fatigue. Chest radiography shows a cavitation of the
right upper lobe. She has hypertension for which she
is taking lisinopril 10 mg/day. She has no known drug
allergies. Which one of the following would be best
for this patient?

53. Answer: D
Because of rifampins intense induction effects on
many drugs metabolized by the cytochrome P450 (CYP)
enzyme system, it should be used with caution. In particular, a patient on concomitant ART should not be prescribed rifampin. Rifabutin is a less potent enzyme inducer
than rifampin and is the preferred agent in patients who
are receiving protease inhibitors (PIs) or nonnucleoside
reverse transcriptase inhibitors (NNRTIs) (Answer D is
correct). Rifamycins and isoniazid should always be firstline treatment of TB. Therefore, replacing rifampin with
moxifloxacin is inappropriate (Answer A is incorrect). The
patient is stable on his current antiretroviral regimen; thus,
discontinuing antiretrovirals is inappropriate (Answer B is
incorrect). Rifapentine is a long-acting rifamycin that can
be used once weekly in the continuation phase of treatment
(after the first 2 months) only for selected HIV-negative
patients and is not an option for this patient. Unlike rifabutin, rifapentine does not offer any advantage in sparing the
drug interactions (Answer C is incorrect).
1. Centers for Disease Control and Prevention. Managing Drug Interactions in the Treatment of HIV-Related
Tuberculosis. Available at www.cdc.gov/tb/publications/
guidelines/TB_HIV_Drugs/default.htm. Accessed April
9, 2012.
2. Abdool Karim SS, Naidoo K, Grobler A, Padayatchi N, Baxter C, Gray A, et al. Timing of initiation of

A. Continue current treatment and check serum


concentrations of her TB drugs.
B. Add ethambutol and check serum concentrations
of her TB drugs.
C. Extend her current TB treatment duration to 12
months.
D. Add ethambutol and moxifloxacin and check
serum concentrations of her TB drugs.
52. Answer: D
Adding ethambutol and moxifloxacin at this point
would be the best option because the patient is having an
insufficient response. The patient is still reporting increased
cough and fatigue, and radiography shows cavitation. Serum
drug concentrations should be considered in patients not
responding to therapy (Answer D is correct). Adding antibiotics to her current regimen is necessary in addition to
checking serum concentrations (Answer A is incorrect). A
single drug should never be added to a failing regimen, so
adding ethambutol alone would be inappropriate (Answer
B is incorrect). The treatment duration in this patient will
PSAP-VII Infectious Diseases

Discontinue rifampin and add moxifloxacin.


Discontinue antiretrovirals for 2 months.
Change rifampin to rifapentine.
Change rifampin to rifabutin.

21

Answers

55. Answer: D

The basic principles that underlie treatment of pulmonary disease also apply to extrapulmonary disease. There
are limited clinical data pertaining to extrapulmonary disease and the therapy duration. Current guidelines suggest 9
months for bone disease (Answer D is correct). Continuing
therapy for 12 months is not necessary because the patient
is showing clinical improvement at follow-up (Answer A
and Answer B are incorrect). Six months of therapy is inadequate for the treatment of osteomyelitis caused by TB
(Answer C is incorrect).
1. Medical Research Council Working Party on Tuberculosis of the Spine. Five-year assessment of controlled
trials of short-course chemotherapy regimens of 6, 9 or
18 months duration for spinal tuberculosis in patients
ambulatory from the start or undergoing radical surgery.
Int Orthop 1999;23:7381.
PubMed Link
2. Dutt AK, Moers D, Stead WW. Short-course chemotherapy for extrapulmonary tuberculosis. Nine years experience. Ann Intern Med 1986;104:712.
PubMed Link

antiretroviral drugs during tuberculosis therapy. N Engl


J Med 2010;362:697706.
PubMed Link
54. Six weeks ago, a 40-year-old man was exposed to a person with active TB. He has been identified as part of a
contact investigation. He is HIV negative but has lost
his job and is about to be evicted from his apartment.
Which one of the following is the best method to
assess this patients infection status?
A. PPD test.
B. IGRA.
C. PPD and sputum sample.
D. IGRA and sputum sample.
54. Answer: B
This patient could be tested with either the skin test
or an IGRA. He is HIV-negative, so response to either test
should be good. However, he has an unstable living situation that is consuming his attention, and he may not be
available for the reading of the skin test. Therefore, obtaining an IGRA is preferred to obtaining a PPD test (Answer
B is correct; Answer A is incorrect). At this stage, sputum testing is not warranted (Answer C and Answer D are
incorrect).
1. Mazurek GH, Jereb J, Vernon A, LoBue P, Goldberg S,
Castro K, et al. Updated guidelines for using interferon
gamma release assays to detect Mycobacterium tuberculosis infectionUnited States, 2010. MMWR Recomm
Rep 2010;59(RR-5):125.
PubMed Link
2. Targeted tuberculin skin testing and treatment of latent
tuberculosis infection. Am J Respir Crit Care Med
2000;161(4 pt 2):S221S247.
PubMed Link

56. A 30-year-old HIV-positive man presents with a PPD


as the result of 5 mm of induration. He feels well, and
physical examination and chest radiography are both
normal. He is not producing sputum and does not have
obvious TB lesions. His CD4+ count is 275 cells/mm3.
He received the bacillus Calmette-Gurin vaccine during childhood. He is currently on antiretroviral therapy
with tenofovir, lamivudine, darunavir, and ritonavir.
Which one of the following is best for this patient?
A. Isoniazid for 12 months.
B. Isoniazid for 9 months.
C. Isoniazid and rifampin for 9 months.
D. Isoniazid and rifampin for 12 months.

55. A 22-year-old woman who received a diagnosis of


osteomyelitis caused by TB has been treated for 2
months with rifampin, isoniazid, pyrazinamide, and
ethambutol. The laboratory reports the isolate is fully
drug susceptible, and she shows clinical improvement
at follow-up. Which one of the following is the best
option for this patient?
A. Discontinue pyrazinamide and ethambutol
and complete 10 more months of isoniazid and
rifampin.
B. Continue all four drugs to complete 12 months
total.
C. Discontinue ethambutol and complete 4 more
months of rifampin, isoniazid, and pyrazinamide.
D. Discontinue pyrazinamide and ethambutol
and complete 7 more months of rifampin and
isoniazid.

Answers

56. Answer: B
Latent TB infection (LTBI) treatment is effective
in preventing active disease in people who have positive
tuberculin skin tests and who are at risk of reactivated TB.
The recommended regimen for all adults is daily isoniazid
for 9 months (Answer B is correct). Secondary or alternative therapies include isoniazid for 6 months; less strongly
recommended is 4 months of rifampin. Randomized, prospective trials support using LTBI treatment in HIV-positive and HIV-negative people. Twelve months of isoniazid
will work, but this is not needed (Answer A is incorrect).
Treatment with isoniazid and rifampin is not indicated
because the patient does not have active disease (Answer C
and Answer D are incorrect).
1. Akolo C, Adetifa I, Shepperd S, Volmink J. Treatment
of latent tuberculosis infection in HIV infected persons.
Cochrane Database Syst Rev 2010;1:CD000171.
22

PSAP-VII Infectious Diseases

PubMed Link
2. Centers for Disease Control and Prevention. Guidelines
for prevention and treatment of opportunistic infections in HIV-infected adults and adolescents. MMWR
2009;58:RR-4. Available at www.cdc.gov/mmwr/pdf/rr/
rr5804.pdf. Accessed April 9, 2012.

A. Continue current treatment for another 7


months.
B. Complete an additional 4 months of isoniazid and
a rifamycin.
C. Complete an additional 6 months of isoniazid and
a rifamycin.
D. Continue current treatment for another 2
months.

Questions 57 and 58 pertain to the following case.


R.P. is a 43-year-old HIV-positive man from Africa who
presents to the clinic with increased weight loss, night
sweats, and cough. He is currently on antiretroviral therapy
with efavirenz, emtricitabine, and tenofovir. His sputum
sample is positive for AFB.

58. Answer: B

The recommended regimen for susceptible TB in HIVinfected adults is a 6-month regimen consisting of an initial
phase of isoniazid and a rifamycin, pyrazinamide, and ethambutol for the first 2 months, followed by a continuation
phase of isoniazid and a rifamycin for 4 months. Although
some clinicians elect to extend treatment of TB for HIVpositive patients, in this case, the patient is responding well,
and at this point, there is no reason to extend the treatment
beyond 6 months (Answer B is correct). Continuing current
treatment for another 7 months is unnecessary (Answer A
is incorrect), as is continuing with isoniazid and a rifamycin
for 6 additional months (Answer C is incorrect). Continuing current treatment for another 2 months would provide
only 4 months of therapy (Answer D is incorrect).
1. Centers for Disease Control and Prevention. Guidelines
for prevention and treatment of opportunistic infections in HIV-infected adults and adolescents. MMWR
2009;58:RR-4. Available at www.cdc.gov/mmwr/pdf/rr/
rr5804.pdf. Accessed April 9, 2012.
2. Swaminathan S, Narendran G, Venkatesan P, Iliayas
S, Santhanakrishnan R, Menon PA, et al. Efficacy of a
6-month versus 9-month intermittent treatment regimen
in HIV-infected patients with tuberculosis: a randomized
clinical trial. Am J Respir Crit Care Med 2010;181:743.
PubMed Link

57. Which one of the following is the best option to


treat R.P.?
A. Isoniazid, rifampin, and pyrazinamide; increase
the dose of efavirenz.
B. Isoniazid, rifampin, pyrazinamide, and ethambutol; no change in antiretroviral therapy.
C. Isoniazid, rifampin, pyrazinamide, and ethambutol; increase the dose of efavirenz.
D. Isoniazid, lower-dose rifabutin, pyrazinamide, and
ethambutol.
57. Answer: B
Efavirenz-based ART and TB drug interactions are
well characterized. Rifampin has minimal effects on efavirenz. Dosage changes are not necessary for efavirenz or
rifampin (Answer B is correct). Isoniazid, rifampin, and
pyrazinamide are inappropriate because a four-drug regimen is required for initial therapy (Answer A is incorrect). The dose of efavirenz need not be increased because
of the minimal effect of rifampin on efavirenz concentrations (Answer C is incorrect). If rifabutin is used with this
combination, the rifabutin dose should be increased, not
decreased (Answer D is incorrect).
1. Centers for Disease Control and Prevention. Managing Drug Interactions in the Treatment of HIV-Related
Tuberculosis. Available at www.cdc.gov/tb/publications/
guidelines/TB_HIV_Drugs/default.htm. Accessed April
9, 2012.
2. Boulle A, Van Cutsem G, Cohen K, Hilderbrand K,
Mathee S, Abrahams M, et al. Outcomes of nevirapineand efavirenz-based antiretroviral therapy when coadministered with rifampicin-based antitubercular therapy.
JAMA 2008;300:5309.
PubMed Link

59. A 74-year-old man has been on therapy with isoniazid,


rifampin, ethambutol, and pyrazinamide for 6 weeks
for pulmonary TB. His isolate is found to be MDR TB.
The patients susceptibility tests indicate his organism
is susceptible to amikacin, ethambutol, levofloxacin,
cycloserine, and p-aminosalicylic acid. His estimated
creatinine clearance is 25 mL/minute, and he has a history of psychosis. Which one of the following is the
best treatment option for this patient?
A. Amikacin, ethambutol, levofloxacin, and p-aminosalicylic acid.
B. Amikacin, cycloserine, and levofloxacin.
C. Cycloserine, ethambutol, levofloxacin, and p-aminosalicylic acid.
D. Cycloserine, levofloxacin, and p-aminosalicylic
acid.

58. Eight weeks later, R.P.s susceptibility report indicates


that his isolate is sensitive to all first-line agents. The
patient is no longer symptomatic and feels well. Which
one of the following is the most appropriate plan
for R.P.?
PSAP-VII Infectious Diseases

59. Answer: A

23

Answers

Based on the sensitivity information, the history of


psychosis, and the patients estimated creatinine clearance,
the best option is amikacin, ethambutol, levofloxacin, and
p-aminosalicylic acid (Answer A is correct). Doses for the
renally cleared drugs amikacin, ethambutol, and levofloxacin would need to be adjusted. Cycloserine should be
avoided in patients with a history of psychological disorders and in patients with a creatinine clearance less than 50
mL/minute. Therefore, Answer B, Answer C, and Answer
D are incorrect because they all include this drug.

2. Targeted tuberculin skin testing and treatment of latent


tuberculosis infection. Am J Respir Crit Care Med
2000;161(4 pt 2):S221S247.
PubMed Link

1. World Health Organization. WHO guidelines for the


programmatic management of drug-resistant tuberculosis:
2011 update. Eur Respir J 2011;38:51628.
PubMed Link
2. Mitnick CD, Shin SS, Seung KJ, Rich ML, Atwood SS,
Furin JJ, et al. Comprehensive treatment of extensively drugresistant tuberculosis. N Engl J Med 2008;359:56374.
PubMed Link
60. A 41-year-old psychiatrist was notified of a TB exposure on the psychiatric unit 10 weeks ago. Last week, a
PPD test was placed and read as negative. She has systemic lupus erythematosus and takes prednisone 15
mg/day and calcium supplements. Her baseline laboratory tests show an AST of 42 IU/L, ALT of 41 IU/L,
and SCr 0.8 mg/dL. Which one of the following is
the best plan for this patient?
A.
B.
C.
D.

Obtain an IGRA.
Place another PPD.
Obtain sputum culture.
Obtain chest radiography.

60. Answer: B
A negative PPD result may indicate that a person has
not been exposed to TB, that the person is not infected with
TB, that his or her immune system has not responded to
the antigen in the test, or that it is too early to detect exposure. It has been more than 6 weeks since the patient was
exposed. To confirm a negative or indeterminate result,
the best option at this time would be to place another
PPD (Answer B is correct). It is too late to obtain an IGRA
because they should be performed no more than 3 days after
a PPD is placed (Answer A is incorrect). A sputum culture
or chest radiography is unnecessary at this time because the
patient is not symptomatic (Answer C and Answer D are
incorrect).
1. Mazurek GH, Jereb J, Vernon A, LoBue P, Goldberg S,
Castro K, et al. Updated guidelines for using interferon
gamma release assays to detect Mycobacterium tuberculosis infectionUnited States, 2010. MMWR Recomm
Rep 2010;59(RR-5):125.
PubMed Link

Answers

24

PSAP-VII Infectious Diseases

Anda mungkin juga menyukai